Tải bản đầy đủ (.pdf) (54 trang)

Tuyển tập những bài toán Hình học phẳng hay và khó ôn thi HSG quốc gia

Bạn đang xem bản rút gọn của tài liệu. Xem và tải ngay bản đầy đủ của tài liệu tại đây (1.05 MB, 54 trang )

<span class='text_page_counter'>(1)</span><div class='page_container' data-page=1>

<b>TUYỂN TẬP NHỮNG BÀI HÌNH HAY ƠN THI HSG QUỐC GIA </b>


<b>1. Cho tam giác 𝑨𝑩𝑪 nội tiếp trong đường tròn (𝑶). 𝑬 là một điểm di động trên (𝑶). 𝑨𝑬 cắt </b>
<b>các tiếp tuyến tại </b>𝑩, 𝑪 của (𝑶) tương ứng tại 𝑴, 𝑵. 𝑩𝑵 cắt 𝑪𝑴 tại 𝑭. Chứng minh rằng
<b>đường thẳng 𝑬𝑭 luôn đi qua một điểm cố định khi 𝑬 di động trên (𝑶). </b>


<i>Lời giải: </i>


Đặt 𝑇 ≡ 𝐴𝐸 ∩ 𝐵𝐶, 𝐽 ≡ 𝐶𝐸 ∩ 𝐴𝐵, 𝐴<sub>1</sub> ≡ 𝐵𝑀 ∩ 𝐶𝑁, 𝐿 ≡ 𝐴<sub>1</sub>𝐹 ∩ 𝐵𝐶, 𝐻 ≡ 𝐸𝐹 ∩ 𝐵𝐶, 𝐾 ≡ 𝐴𝐹 ∩ 𝐵𝐶.
Xét tứ giác toàn phần sinh bởi bốn điểm 𝐵, 𝐶, 𝑀, 𝑁 ta có (𝑇𝐿𝐵𝐶) = −1, hơn nữa cực 𝐴1 của 𝐵𝐶


đối với đường tròn (𝑂) thuộc đường thẳng 𝐴̅̅̅̅̅̅̅1𝐹𝐿 nên cực của 𝐴̅̅̅̅̅̅̅1𝐹𝐿 thuộc đường thẳng 𝐵𝐶, suy


ra 𝑇 chính là cực của đường thẳng 𝐴̅̅̅̅̅̅̅1𝐹𝐿 đối với (𝑂). Hơn nữa, xét tứ giác toàn phần tạo bởi bốn


điểm 𝐴, 𝐸, 𝐵, 𝐶 (thuộc (𝑂)) thì ta thấy 𝑇 và 𝐽 cũng liên hợp với nhau qua (𝑂), suy ra 𝐽 ∈ 𝐴̅̅̅̅̅̅̅<sub>1</sub>𝐹𝐿 –
đối cực của 𝑇. Nói cách khác ta có 𝐴1, 𝐹, 𝐿, 𝐽 thẳng hàng.


Mặt khác đặt 𝐴<sub>1</sub>′ ≡ 𝐸𝐾 ∩ 𝐴𝐻. Xét tứ giác toàn phần sinh bởi bốn điểm 𝐴, 𝐸, 𝐾, 𝐻 thì ta lại có
(𝐹𝑇, 𝐹𝐴<sub>1</sub>′<sub>, 𝐹𝐸, 𝐹𝐴) = −1 nên </sub><sub>𝐴</sub>


1
′ <sub>∈ 𝐴</sub>


1𝐹𝐿𝐽


̅̅̅̅̅̅̅̅<sub>. Hơn nữa (𝑇𝐹, 𝑇𝐴</sub><sub>1</sub>′<sub>, 𝑇𝐸, 𝑇𝐵) = −1 = 𝑇(𝐹𝐴</sub>


1𝐸𝐵) nên


𝐴<sub>1</sub>′ ≡ 𝐴<sub>1</sub>. Do vậy 𝐻 ≡ 𝐸𝐹 ∩ 𝐵𝐶 ≡ 𝐴𝐴<sub>1</sub>∩ 𝐵𝐶, cố định.



L
K


J


T H


A<sub>1</sub>
F


N


M


A


B C


</div>
<span class='text_page_counter'>(2)</span><div class='page_container' data-page=2>

Tóm lại 𝐸𝐹 đi qua điểm 𝐻 cố định, là chân đường đối trung từ 𝐴 của tam giác 𝐴𝐵𝐶. Kết thúc
chứng minh.


<b>2. Trong mặt phẳng ta xét bốn điểm </b>𝑨, 𝑩, 𝑪, 𝑫 bất kỳ. Từ mỗi điểm trong chúng, ta vẽ các
<b>hình chiếu đến các cạnh của tam giác tạo bởi ba điểm cịn lại, đó là các tam giác pedal. Khi </b>
<b>đó, đường trịn ngoại tiếp bốn tam giác pedal (ứng với bốn điểm ấy) đồng quy tại một điểm. </b>


<i>Lời giải:</i>


Trước hết ta chứng minh bổ đề sau:


<b>Bổ đề 1. Cho hai đường tròn (𝑂</b><sub>1</sub>) và (𝑂<sub>2</sub>) cắt nhau tại hai điểm 𝐴, 𝐵. Hai đường thẳng qua 𝐵,


một đường cắt (𝑂1), (𝑂2) tại 𝑀, 𝑁 và đường kia cắt (𝑂1), (𝑂2) tại 𝑃, 𝑄. Khi đó 𝐴 là tâm của phép


đồng dạng thuận biến 𝑀𝑁 thành 𝑃𝑄.
<b>Chứng minh. </b>


Từ các bộ điểm đồng viên ta có:


(𝐴𝑀, 𝐴𝑃) ≡ (𝐵𝑀, 𝐵𝑃) ≡ (𝐴𝑁, 𝐴𝑄) (𝑚𝑜𝑑 𝜋)
(𝑀𝐴, 𝑀𝑁) ≡ (𝑃𝐴, 𝑃𝑄) (𝑚𝑜𝑑 𝜋)
Nên tam giác 𝐴𝑀𝑁 và tam giác 𝐴𝑃𝑄 đồng dạng thuận với nhau.
Từ đó suy ra 𝐴 là tâm phép đồng dạng thuận biến 𝑀𝑁 thành 𝑃𝑄.


Had


Ad


Ac


Ab


S


Cb


Cd


Ca


Dc



Da


Db


A


B C


</div>
<span class='text_page_counter'>(3)</span><div class='page_container' data-page=3>

Vào bài.


Với điểm 𝐴, ta sẽ gọi hình chiếu của nó tương ứng lên cạnh không chứa 𝐵, 𝐶, 𝐷 của tam giác
𝐵𝐶𝐷 là 𝐴<sub>𝑏</sub>, 𝐴<sub>𝑐</sub>, 𝐴<sub>𝑑</sub> và tương tự cho các điểm cịn lại.


Ta có 𝐴, 𝐴𝑑, 𝐴𝑏, 𝐶 đồng viên cũng như 𝐴, 𝐴𝑑, 𝐴𝑐, 𝐵 đồng viên nên:


(𝐴<sub>𝑑</sub>𝐴<sub>𝑏</sub>, 𝐴<sub>𝑑</sub>𝐴<sub>𝑐</sub>) ≡ (𝐴<sub>𝑑</sub>𝐴<sub>𝑏</sub>, 𝐴<sub>𝑑</sub>𝐴) + (𝐴<sub>𝑑</sub>𝐴, 𝐴<sub>𝑑</sub>𝐴<sub>𝑐</sub>) ≡ (𝐶𝐷, 𝐶𝐴) + (𝐵𝐷, 𝐵𝐴)
≡ (𝐴𝐶, 𝐴𝐵) + ((𝐵𝐷, 𝐵𝐴) + (𝐴𝐵, 𝐴𝐶) + (𝐴𝐶, 𝐶𝐷))
≡ (𝐴𝐶, 𝐴𝐵) + (𝐷𝐵, 𝐷𝐶) (𝑚𝑜𝑑 𝜋)


Lý luận tương tự ta có:


(𝐴<sub>𝑑</sub>𝐴<sub>𝑐</sub>, 𝐴<sub>𝑑</sub>𝐴<sub>𝑏</sub>) ≡ (𝐵<sub>𝑐</sub>𝐵<sub>𝑑</sub>, 𝐵<sub>𝑐</sub>𝐵<sub>𝑎</sub>) ≡ (𝐶<sub>𝑏</sub>𝐶<sub>𝑎</sub>, 𝐶<sub>𝑏</sub>𝐶<sub>𝑑</sub>) ≡ (𝐷<sub>𝑎</sub>𝐷<sub>𝑏</sub>, 𝐷<sub>𝑎</sub>𝐷<sub>𝑐</sub>)
≡ (𝐴𝐵, 𝐴𝐶) + (𝐷𝐶, 𝐷𝐵) (𝑚𝑜𝑑 𝜋)


Điều tương tự cũng xảy ra với các góc cịn lại nên ta có các tam giác pedal ứng với các điểm
𝐴, 𝐵, 𝐶, 𝐷 đồng dạng thuận với nhau.


Mặt khác ta có 𝐴<sub>𝑏</sub>, 𝐴<sub>𝑐</sub>, 𝐷<sub>𝑏</sub>, 𝐷<sub>𝑐</sub> đều thuộc đường trịn đường kính 𝐴𝐷 nên theo định lý Pascal thì
𝐵 ≡ 𝐴𝐷𝑐∩ 𝐷𝐴𝑐, 𝐶 ≡ 𝐴𝐷𝑏∩ 𝐷𝐴𝑏, 𝐻𝑎𝑑 ≡ 𝐴𝑏𝐷𝑐 ∩ 𝐴𝑐𝐷𝑏 thẳng hàng.



Hơn nữa ta đã chứng minh hai tam giác 𝐴<sub>𝑑</sub>𝐴<sub>𝑏</sub>𝐴<sub>𝑐</sub> và 𝐷<sub>𝑎</sub>𝐷<sub>𝑐</sub>𝐷<sub>𝑏</sub> đồng dạng thuận nên:
(𝐻𝑎𝑑𝐴𝑏, 𝐻𝑎𝑑𝐴𝑐) ≡ (𝐴𝑏𝐷𝑐, 𝐴𝑐𝐷𝑏) ≡ (𝐴𝑑𝐴𝑏, 𝐴𝑑𝐴𝑐) (𝑚𝑜𝑑 𝜋)


Suy ra 𝐻<sub>𝑎𝑑</sub>, 𝐴<sub>𝑏</sub>, 𝐴<sub>𝑐</sub>, 𝐴<sub>𝑑</sub> đồng viên cũng như 𝐻<sub>𝑎𝑑</sub>, 𝐷<sub>𝑎</sub>, 𝐷<sub>𝑏</sub>, 𝐷<sub>𝑐</sub> đồng viên.


Nói cách khác thì 𝐴𝑑𝐷𝑎, 𝐴𝑏𝐷𝑐, 𝐴𝑐𝐷𝑏 đồng qui tại 𝐻𝑎𝑑 là một trong hai giao điểm của hai đường


tròn pedal ứng với 𝐴 và 𝐷. Gọi 𝑆<sub>𝑎𝑑</sub> là giao điểm còn lại của hai đường tròn pedal này, theo bổ đề
thì 𝑆<sub>𝑎𝑑</sub> chính là tâm của phép đồng dạng thuận (phép vị tự quay) biến Δ𝐴<sub>𝑑</sub>𝐴<sub>𝑏</sub>𝐴<sub>𝑐</sub> ↦ Δ𝐷<sub>𝑎</sub>𝐷<sub>𝑐</sub>𝐷<sub>𝑏</sub>.
Bây giờ gọi 𝑆𝑎𝑐, 𝑆𝑐𝑑 là giao điểm của hai đường tròn pedal ứng với 𝐴 và 𝐶,𝐶 và 𝐷 sao cho 𝑆𝑎𝑐 và


𝑆𝑐𝑑 không nằm trên bất cứ đường thẳng nào của tứ giác toàn phần sinh bởi bốn điểm 𝐴, 𝐵, 𝐶, 𝐷.


Bằng lý luận tương tự như trên ta suy ra 𝑆<sub>𝑎𝑐</sub>, 𝑆<sub>𝑐𝑑</sub> là tâm vị tự quay biến Δ𝐴<sub>𝑑</sub>𝐴<sub>𝑏</sub>𝐴<sub>𝑐</sub> ↦ 𝐶<sub>𝑏</sub>𝐶<sub>𝑑</sub>𝐶<sub>𝑎</sub> và
Δ𝐶<sub>𝑏</sub>𝐶<sub>𝑑</sub>𝐶<sub>𝑎</sub> ↦ Δ𝐷<sub>𝑎</sub>𝐷<sub>𝑐</sub>𝐷<sub>𝑏</sub>.


N


Q
B


A


O1 O2


M


</div>
<span class='text_page_counter'>(4)</span><div class='page_container' data-page=4>

Rõ ràng tích của hai phép đồng dạng thuận tâm 𝑆<sub>𝑎𝑐</sub> và 𝑆<sub>𝑎𝑑</sub> chính là phép đồng dạng thuận có tâm
là 𝑆<sub>𝑐𝑑</sub>. Mặt khác ta có:



(𝐶<sub>𝑑</sub>𝐷<sub>𝑐</sub>, 𝐷<sub>𝑎</sub>𝐶<sub>𝑏</sub>) ≡ (𝐻<sub>𝑐𝑑</sub>𝐷<sub>𝑐</sub>, 𝐻<sub>𝑐𝑑</sub>𝐷<sub>𝑎</sub>) ≡ (𝐷<sub>𝑏</sub>𝐷<sub>𝑐</sub>, 𝐷<sub>𝑏</sub>𝐷<sub>𝑎</sub>) ≡ (𝐴<sub>𝑐</sub>𝐴<sub>𝑏</sub>, 𝐴<sub>𝑐</sub>𝐴<sub>𝑑</sub>) ≡ (𝐻<sub>𝑎𝑑</sub>𝐴<sub>𝑏</sub>, 𝐻<sub>𝑎𝑑</sub>𝐴<sub>𝑐</sub>)
≡ (𝐴<sub>𝑏</sub>𝐷<sub>𝑐</sub>, 𝐴<sub>𝑑</sub>𝐷<sub>𝑎</sub>) (𝑚𝑜𝑑 𝜋)


Bằng lý luận tương tự thì:


(𝐶<sub>𝑑</sub>𝐷<sub>𝑐</sub>, 𝐷<sub>𝑎</sub>𝐶<sub>𝑏</sub>) ≡ (𝐴<sub>𝑏</sub>𝐷<sub>𝑐</sub>, 𝐴<sub>𝑑</sub>𝐷<sub>𝑎</sub>) ≡ (𝐴<sub>𝑏</sub>𝐶<sub>𝑑</sub>, 𝐴<sub>𝑑</sub>𝐶<sub>𝑏</sub>) (𝑚𝑜𝑑 𝜋)


Suy ra tam giác 𝐴<sub>𝑏</sub>𝐶<sub>𝑑</sub>𝐷<sub>𝑐</sub> và tam giác 𝐴<sub>𝑑</sub>𝐶<sub>𝑏</sub>𝐷<sub>𝑎</sub> cũng đồng dạng thuận. Vậy nên tồn tại một phép
đồng dạng duy nhất 𝑉Δ𝐴<sub>𝑏</sub>𝐶<sub>𝑑</sub>𝐷<sub>𝑐</sub> ↦ Δ𝐴<sub>𝑑</sub>𝐶<sub>𝑏</sub>𝐷<sub>𝑎</sub>. Hơn nữa:


𝑉<sub>𝑎𝑑</sub>: 𝐴<sub>𝑑</sub>𝐴<sub>𝑏</sub> ↦ 𝐷<sub>𝑎</sub>𝐷<sub>𝑐</sub>, 𝑉<sub>𝑎𝑐</sub>: 𝐶<sub>𝑏</sub>𝐶<sub>𝑑</sub> ↦ 𝐴<sub>𝑑</sub>𝐴<sub>𝑏</sub>, 𝑉<sub>𝑐𝑑</sub>= 𝑉<sub>𝑎𝑑</sub>⋅ 𝑉<sub>𝑎𝑐</sub>: 𝐶<sub>𝑏</sub>𝐶<sub>𝑑</sub> ↦ 𝐷<sub>𝑎</sub>𝐷<sub>𝑐</sub>
Nên: 𝑆<sub>𝑎𝑑</sub> ≡ 𝑆<sub>𝑎𝑐</sub> ≡ 𝑆<sub>𝑎𝑑</sub> ≡ 𝑆.


Lý luận tương tự thì ta thấy các đường tròn pedal của 𝐴, 𝐵, 𝐶, 𝐷 đều đi qua 𝑆 (đpcm).


<b>3. (VMO 2009) Trong mặt phẳng cho hai điểm phân biệt </b>𝑨, 𝑩 và một điểm 𝑪 di động sao
<b>cho 𝑨𝑪𝑩</b>̂ = 𝒂 khơng đổi. Đường trịn (𝑰) nội tiếp tam giác 𝑨𝑩𝑪 tiếp xúc với 𝑨𝑩, 𝑩𝑪, 𝑪𝑨
<b>theo thứ tự tại 𝑫, 𝑬, 𝑭. 𝑨𝑰, 𝑩𝑰 cắt 𝑬𝑭 theo thứ tự tại 𝑴, 𝑵 </b>


<b>a. Chứng minh rằng 𝑴𝑵 có độ dài khơng đổi. </b>


<b>b. Chứng minh rằng đường tròn ngoại tiếp tam giác 𝑫𝑴𝑵 đi qua một điểm cố định. </b>


<i>Lời giải: </i>


Hcd


Had


Ad



Ac


Ab


S


Cb


Cd


Dc


Da


Db


B


</div>
<span class='text_page_counter'>(5)</span><div class='page_container' data-page=5>

a. Ta có:


(𝐼𝐴, 𝐼𝐵) ≡ (𝐼𝐴, 𝐴𝐵) + (𝐴𝐵, 𝐼𝐵) ≡1


2((𝐶𝐴, 𝐴𝐵) + (𝐴𝐵, 𝐵𝐶)) ≡
1


2(𝐶𝐴, 𝐶𝐵) (𝑚𝑜𝑑 𝜋)
Hơn nữa:


(𝐹𝐶, 𝐹𝐸) ≡ (𝐹𝐶, 𝐶𝐸) + (𝐶𝐸, 𝐹𝐸) ≡ (𝐹𝐶, 𝐶𝐸) + (𝐹𝐸, 𝐹𝐶) (𝑚𝑜𝑑 𝜋)


Nên:


(𝐹𝐶, 𝐹𝐸) ≡1


2(𝐹𝐶, 𝐶𝐸) ≡
1


2(𝐶𝐴, 𝐶𝐵) (𝑚𝑜𝑑 𝜋)
Suy ra:


(𝐼𝐴, 𝐼𝐵) ≡ (𝐹𝐶, 𝐹𝐸) (𝑚𝑜𝑑 𝜋)
Từ đó: 𝐼, 𝐴, 𝑁, 𝐹 đồng viên, nên:


(𝑁𝐴, 𝑁𝐼) ≡ (𝐹𝐴, 𝐹𝐼) ≡𝜋


2 (𝑚𝑜𝑑 𝜋)
Lý luận tương tự ta cũng có:


(𝑀𝐵, 𝑀𝐼) ≡ (𝑁𝐴, 𝑁𝐼) ≡𝜋


2 (𝑚𝑜𝑑 𝜋)


Suy ra 𝑀, 𝑁 thuộc đường trịn đường kính 𝐴𝐵. Gọi 𝑆 là trung điểm của 𝐴𝐵 thì từ các bộ điểm
đồng viên ta có:


(𝑆𝑀, 𝑆𝑁) ≡ 2(𝐵𝑀, 𝐵𝑁) ≡ 2(𝐸𝑀, 𝐸𝐼) ≡ 2(𝐶𝐹, 𝐶𝐼) ≡ (𝐶𝐴, 𝐶𝐵) (𝑚𝑜𝑑 𝜋)
Suy ra góc hình học 𝑀𝑆𝑁̂ có độ lớn khơng đổi.


Mặt khác theo định lý sin thì:



𝑀𝑁 = 𝐴𝐵


2 ⋅ sin 𝑀𝑆𝑁̂
Nên 𝑀𝑁 có độ lớn khơng đổi (đpcm).


H


S
M


N
F


E


D
I


O
C


</div>
<span class='text_page_counter'>(6)</span><div class='page_container' data-page=6>

b. Gọi 𝐻 ≡ 𝐴𝑁 ∩ 𝐵𝑀. Xét tam giác 𝐻𝐴𝐵 có 𝑀, 𝐷, 𝑁 là chân các đường cao (theo các lý luận ở
câu a). Thế thì (𝐷𝑀𝑁) là đường tròn Euler của tam giác 𝐻𝐴𝐵, và do đó, nó đi qua trung điểm 𝑆
của cạnh 𝐴𝐵, là một điểm cố định (đpcm).


<b>4. Cho tam giác </b>𝑨𝑩𝑪 có 𝑨𝑨′, 𝑩𝑩′, 𝑪𝑪′ là các đường cao. Chứng minh rằng đường thẳng
<b>Euler của các tam giác 𝑨𝑩</b>′𝑪′, 𝑩𝑪′𝑨′, 𝑪𝑨′𝑩′<b> đồng qui. </b>


<i>Lời giải:</i>



Gọi 𝑂<sub>𝑎</sub>, 𝑂<sub>𝑏</sub>, 𝑂<sub>𝑐</sub> là tâm đường tròn ngoại tiếp các tam giác 𝐴𝐵<i>′</i>𝐶<i>′</i>, 𝐵𝐶<i>′</i>𝐴<i>′</i>, 𝐶𝐴<i>′</i>𝐵<i>′</i> còn 𝐻<sub>𝑎</sub>, 𝐻<sub>𝑏</sub>, 𝐻<sub>𝑐</sub> là
trực tâm của các tam giác ấy theo thứ tự.


Xét phép vị tự đối xứng 𝑆𝑎 có trục đối xứng là phân giác 𝑙𝑎 của góc 𝐵𝐴𝐶̂, tâm vị tự 𝐴 và tỉ số:


𝑘 =𝐴𝐵


<i>′</i>


𝐴𝐵 = cos 𝐴


Ta có 𝑆<sub>𝑎</sub>: 𝐵 ↦ 𝐵<i>′</i>, 𝐶 ↦ 𝐶<i>′</i>, 𝑂 ↦ 𝑂<sub>𝑎</sub>, 𝐻 ↦ 𝐻<sub>𝑎</sub> nên 𝑂<sub>𝑎</sub>𝐻<sub>𝑎</sub> và 𝑂𝐻 đối song với nhau trong góc 𝐵𝐴𝐶̂.
Gọi 𝐼 là tâm đường tròn nội tiếp tam giác 𝐴𝐵𝐶, ta biết rằng tích của hai phép đối xứng trục
𝐼𝐴, 𝐼𝐵 là phép quay tâm 𝐼, góc quay 2(𝐼𝐴, 𝐼𝐵) (𝑚𝑜𝑑 𝜋) nên:


(𝑂𝑎𝐻𝑎, 𝑂𝑏𝐻𝑏) ≡ 2(𝐼𝐴, 𝐼𝐵) ≡ 2((𝐼𝐴, 𝐴𝐵) + (𝐴𝐵, 𝐼𝐵)) ≡ (𝐶𝐴, 𝐴𝐵) + (𝐴𝐵, 𝐵𝐶)


≡ (𝐶𝐴, 𝐶𝐵) (𝑚𝑜𝑑 𝜋)


Mặt khác, 𝑂<sub>𝑐</sub>𝑂<sub>𝑎</sub> và 𝑂<sub>𝑐</sub>𝑂<sub>𝑏</sub> là đường trung bình của tam giác 𝐻𝐴𝐶 và tam giác 𝐻𝐴𝐵 nên:
(𝑂<sub>𝑐</sub>𝑂<sub>𝑎</sub>, 𝑂<sub>𝑐</sub>𝑂<sub>𝑏</sub>) ≡ (𝐶𝐴, 𝐶𝐵) ≡ (𝑂<sub>𝑎</sub>𝐻<sub>𝑎</sub>, 𝑂<sub>𝑏</sub>𝐻<sub>𝑏</sub>) (𝑚𝑜𝑑 𝜋)


I


O<sub>b</sub>
H<sub>b</sub>


S


O<sub>c</sub>


H<sub>c</sub>
H<sub>a</sub>


O<sub>a</sub>


A'


B'


C'


H


O
A


</div>
<span class='text_page_counter'>(7)</span><div class='page_container' data-page=7>

Suy ra giao điểm 𝑆𝑐 của 𝑂𝑎𝐻𝑎 với 𝑂𝑏𝐻𝑏 nằm trên đường tròn (𝑂𝑎𝑂𝑏𝑂𝑐), tức là đường tròn Euler


của tam giác 𝐴𝐵𝐶.


Lý luận tương tự thì ta có 𝑆<sub>𝑎</sub> ≡ 𝑆<sub>𝑏</sub> ≡ 𝑆<sub>𝑐</sub> ≡ 𝑆 ∈ (Σ), hay 𝑂<sub>𝑎</sub>𝐻<sub>𝑎</sub>, 𝑂<sub>𝑏</sub>𝐻<sub>𝑏</sub>, 𝑂<sub>𝑐</sub>𝐻<sub>𝑐</sub> đồng qui trên đường
trịn (Σ), trong đó (Σ) là đường tròn Euler của tam giác 𝐴𝐵𝐶 (đpcm).


<b>5. Cho tam giác 𝑨𝑩𝑪 và 𝑫 là một điểm sao cho 𝑫𝑩𝑨</b>̂ = 𝑫𝑪𝑨̂ = 𝑩𝑨𝑪̂<b>. Chứng minh rằng 𝑫 </b>
<b>thuộc đường thẳng Euler của tam giác 𝑨𝑩𝑪. </b>


<i>Lời giải:</i>


Gọi 𝐾𝑏≡ 𝐶𝐷 ∩ 𝐴𝐵, 𝐾𝑏 ≡ 𝐵𝐷 ∩ 𝐴𝐶, 𝑀𝑏, 𝑀𝑐 là trung điểm của 𝐴𝐵, 𝐴𝐶, 𝐺, 𝑂 lần lượt là trọng tâm



của tam giác 𝐴𝐵𝐶, 𝑇 ≡ 𝐶𝑀<sub>𝑐</sub>∩ 𝐾<sub>𝑏</sub>𝑀<sub>𝑏</sub>, 𝐻 ≡ 𝐶𝐾<sub>𝑏</sub>∩ 𝐾<sub>𝑐</sub>𝑀<sub>𝑐</sub>. Ta có các tam giác 𝐾<sub>𝑏</sub>𝐴𝐶 và 𝐾<sub>𝑐</sub>𝐴𝐵 cân
tương ứng tại 𝐾<sub>𝑏</sub>, 𝐾<sub>𝑐</sub> nên 𝐾<sub>𝑏</sub>, 𝑂, 𝑇, 𝑀<sub>𝑏</sub> thẳng hàng cũng như 𝐾<sub>𝑐</sub>, 𝐻, 𝑂, 𝑀<sub>𝑐</sub> thẳng hàng. Ta lại có:


(𝐶𝑇𝐺𝑀𝑐) = 𝑀𝑏(𝐶𝑇𝐺𝑀𝐶) = 𝑀𝑏(𝐴𝐾𝑏𝐵𝑀𝑐) = 𝐾𝑐(𝐴𝐾𝑏𝐵𝑀𝑐) = 𝐾𝑐(𝐶𝐾𝑏𝐷𝐻) = (𝐶𝐾𝑏𝐷𝐻)


Suy ra 𝑇𝐾𝑏, 𝐺𝐷, 𝐻𝑀𝑐 đồng qui, nói cách khác thì 𝐺𝐷 đi qua giao điểm 𝑂 của 𝐾𝑏𝑀𝑏 và 𝐾𝑐𝑀𝑐,


hay 𝐷 thuộc đường thẳng Euler 𝑂𝐺 của tam giác 𝐴𝐵𝐶 (đpcm).


<b>6. Cho tam giác 𝑨𝑩𝑪 nội tiếp trong đường tròn (𝑶). 𝑺</b>𝟏, 𝑺𝟐<b> là hai điểm di động trên đường </b>


<b>tròn (𝑶) và đối xứng nhau của 𝑶. Giả sử 𝚫</b>𝟏, 𝚫𝟐<b> là các đường thẳng Simson ứng với 𝑺</b>𝟏, 𝑺𝟐<b>. </b>


<b>Chứng minh rằng 𝚫</b><sub>𝟏</sub> ⊥ 𝚫<sub>𝟐</sub><b> cũng như giao điểm của 𝚫</b><sub>𝟏</sub>, 𝚫<sub>𝟐</sub><b> thuộc một đường tròn cố định. </b>


<i>Lời giải:</i>


H
T


G


M<sub>c</sub> <sub>M</sub>


b


D


K<sub>c</sub>



K<sub>b</sub>


O
A


</div>
<span class='text_page_counter'>(8)</span><div class='page_container' data-page=8>

Gọi 𝑀𝑎𝑖, 𝑀𝑏𝑖, 𝑀𝑐𝑖 (𝑖 = 1,2) là hình chiếu của 𝑆𝑖 tương lên 𝐵𝐶, 𝐶𝐴, 𝐴𝐵. Ta có:


(Δ<sub>1</sub>,Δ<sub>2</sub>) ≡ (Δ<sub>1</sub>, 𝐴𝐵) + (𝐴𝐵,Δ<sub>2</sub>) ≡ (𝑆1𝑀𝑎1, 𝑆1𝐵) + (𝑆2𝐴, 𝑆2𝑀𝑏2)


≡ (𝐵𝐶, 𝑆<sub>1</sub>𝐵) +𝜋


2+ (𝑆2𝐴, 𝐴𝐶) +
𝜋


2 ≡ (𝐴𝐶, 𝐴𝑆1) + (𝑆2𝐴, 𝐴𝐶) ≡ (𝑆2𝐴, 𝐴𝑆1)
≡ 𝜋


2 (𝑚𝑜𝑑 𝜋)
Suy ra Δ<sub>1</sub> ⊥Δ<sub>2</sub> tại 𝑆.


Gọi 𝐻 là trực tâm của tam giác 𝐴𝐵𝐶 và (𝐸) là đường tròn Euler của tam giác 𝐴𝐵𝐶. Xét phép vị
tự 𝐻 có tâm 𝐻, tỉ số 𝑘 = 2, ta có:


𝐻:Δ1 ↦Δ1<i>′</i>;Δ2 ↦Δ2<i>′</i> ; 𝑆 ↦ 𝑆<i>′</i>; 𝐸 ↦ 𝑂; (𝐸) ↦ (𝑂)


Thế thì theo tính chất của đường thẳng Steiner, 𝑆1 ∈Δ1<i>′</i> và 𝑆2 ∈Δ2<i>′</i>. Mặt khác do 𝑆1𝑆2 là đường


kính của (𝑂) và Δ1<i>′</i> ⊥Δ2<i>′</i> nên 𝑆<i>′</i>≡ Δ1<i>′</i> ∩Δ2<i>′</i> ∈ (𝑂), mà 𝐻: (𝐸) ↦ (𝑂) nên 𝑆 ∈ (𝐸).


Vậy giao điểm 𝑆 của Δ1,Δ2 thuộc đường tròn Euler của tam giác 𝐴𝐵𝐶 cố định (đpcm).



<b>7. Cho tứ giác 𝑨𝑩𝑪𝑫 nội tiếp trong đường tròn (𝑶) và 𝑴 là một điểm trên đường tròn (𝑶). </b>
<b>Gọi </b> 𝑿, 𝒀, 𝒁, 𝑻, 𝑼, 𝑽 theo thứ tự ấy là hình chiếu của 𝑴 lên các đường thẳng
𝑨𝑩, 𝑩𝑪, 𝑪𝑫, 𝑫𝑨, 𝑨𝑪, 𝑩𝑫 và gọi 𝑵, 𝑷, 𝑸 cũng theo thứ tự ấy là trung điểm của 𝑿𝒁, 𝒀𝑻, 𝑼𝑽.
<b>a. Chứng minh rằng 𝑵, 𝑷, 𝑸 thẳng hàng. </b>


<b>b. Hãy xét trường hợp 𝑴 là một điểm bất kỳ trong mặt phẳng. </b>


<i>Lời giải:</i>


a. Do 𝑀 ∈ (𝑂) nên theo định lý đường thẳng Simson, ta có: (𝑋𝑉𝑇), (𝑋𝑈𝑌), (𝑉𝑍𝑌), (𝑇𝑍𝑈) là các
bộ ba điểm thẳng hàng. Nói cách khác, (𝑋𝑈, 𝑋𝑉, 𝑉𝑌, 𝑇𝑈, 𝑋𝑍, 𝑈𝑉) là một hình tứ giác tồn phần.
Theo định lý về đường thẳng Gauss đối với tứ giác tồn phần thì ta suy ra 𝑁, 𝑃, 𝑄 thẳng hàng.


H


S
E


M<sub>c1</sub>


M<sub>c2</sub>


M<sub>a2</sub>
M<sub>b2</sub>


M<sub>b1</sub>


M<sub>a1</sub>



S<sub>2</sub>


O
A


B


C


</div>
<span class='text_page_counter'>(9)</span><div class='page_container' data-page=9>

b. Nếu 𝑀 là một điểm bất kỳ trong mặt phẳng, ta sẽ chứng minh rằng kết luận của bài toán vẫn
đúng. Thật vậy:


Gọi 𝐺 là trọng tâm của tứ giác 𝐴𝐵𝐶𝐷, tức 𝐺 thỏa hệ thức tỉ cự 𝐺𝐴⃗⃗⃗⃗⃗ + 𝐺𝐵⃗⃗⃗⃗⃗ + 𝐺𝐶⃗⃗⃗⃗⃗ + 𝐺𝐷⃗⃗⃗⃗⃗ = 0⃗ . Khi
đó, 𝐺 chính là điểm đồng qui của đường nối trung điểm hai đường chéo với hai đường nối trung
điểm của hai cặp cạnh đối của tứ giác 𝐴𝐵𝐶𝐷.


P Y


T


Q


U


V


N


Z
X



O


M
A


D


</div>
<span class='text_page_counter'>(10)</span><div class='page_container' data-page=10>

Với điểm 𝑀 bất kỳ trong mặt phẳng, ta gọi 𝑀<i>′</i> ≡ 𝑂𝑀 ∩ (𝑂). Gọi 𝑌<i>′</i>, 𝑇<i>′</i> là hình chiếu của 𝑀<i>′</i> lên
𝐴𝐷, 𝐵𝐶 và 𝑃<i>′</i>, 𝐸, 𝐹 là trung điểm của 𝑌<i>′</i>𝑇<i>′</i>, 𝐵𝐶, 𝐴𝐷 theo thứ tự. Theo định lý Thalès thì ta có:


𝐹𝑇<i>′</i>


̅̅̅̅
𝐹𝑇
̅̅̅̅ =


𝐸𝑌<i>′</i>


̅̅̅̅̅
𝐸𝑌
̅̅̅̅ =


𝑂𝑀<i>′</i>


̅̅̅̅̅
𝑂𝑀
̅̅̅̅̅ = 𝑘
Suy ra 𝐺, 𝑃<i>′</i>, 𝑃 thẳng hàng và:



𝐺𝑃<i>′</i>


̅̅̅̅̅
𝐺𝑃
̅̅̅̅ = 𝑘
Bằng lý luận tương tự với 𝑁, 𝑄 thì ta có:


𝐺𝑁<i>′</i>


̅̅̅̅̅
𝐺𝑁
̅̅̅̅ =


𝐺𝑃<i>′</i>


̅̅̅̅̅
𝐺𝑃
̅̅̅̅ =


𝐺𝑄<i>′</i>


̅̅̅̅̅
𝐺𝑄
̅̅̅̅ = 𝑘


Xét phép vị tự 𝐻 có tâm 𝐺 và tỉ số 𝑘, ta có 𝐻: 𝑁 ↦ 𝑁<i>′</i>, 𝑃 ↦ 𝑃<i>′</i>, 𝑄 ↦ 𝑄<i>′</i> và theo câu a thì 𝑁<i>′</i>, 𝑃<i>′</i>, 𝑄<i>′</i>


thẳng hàng nên 𝑁, 𝑃, 𝑄 cũng thẳng hàng (đpcm).


<b>8. Cho tam giác </b>𝑨𝑩𝑪 có 𝑰 là tâm đường tròn nội tiếp, 𝑶 là tâm đường tròn ngoại tiếp và


<b>trọng tâm 𝑮. Giả sử rằng 𝑶𝑰𝑨</b>̂ = 𝟗𝟎𝒐<b>. Chứng minh rằng 𝑰𝑮 ∥ 𝑩𝑪. </b>


<i>Lời giải: </i>


P
P'
G


F


E


T
T'


Y
Y'


M'
O


M
A


B


C


</div>
<span class='text_page_counter'>(11)</span><div class='page_container' data-page=11>

Ta có:



𝑂𝐼𝐴̂ = 90𝑜<sub>⇔ 𝑂𝐴</sub>2 <sub>= 𝑂𝐼</sub>2<sub>+ 𝐼𝐴</sub>2 <sub>⇔ 𝑅</sub>2 <sub>= 𝑅</sub>2<sub>− 2𝑅𝑟 + 𝐼𝐴</sub>2 <sub>⇔ 𝐼𝐴</sub>2 <sub>= 2𝑅𝑟 </sub>


Mặt khác, đặt 𝑆<sub>𝑎</sub> ≡ 𝐴𝐼 ∩ (𝑂), 𝐿<sub>𝑎</sub> ≡ 𝐴𝐼 ∩ 𝐵𝐶, 𝑀<sub>𝑎</sub> ≡ 𝐴𝐺 ∩ 𝐵𝐶. Do 𝑂𝐼𝐴̂ = 90𝑜<sub> nên: </sub>


𝐼𝐴 = 𝐼𝑆<sub>𝑎</sub> = 𝐴𝑆𝑎


2 ⇒ 𝐴𝑆𝑎


2 <sub>= 4𝐴𝐼</sub>2 <sub>= 8𝑅𝑟 </sub>


Lại có tam giác 𝐴𝐵𝐿<sub>𝑎</sub> và tam giác 𝐴𝑆<sub>𝑎</sub>𝐶 đồng dạng nên:
𝐴𝐵


𝐴𝐿<sub>𝑎</sub> =
𝐴𝑆<sub>𝑎</sub>


𝐴𝐶 ⇒ 𝐴𝐿𝑎⋅ 𝐴𝑆𝑎 = 𝐴𝐵 ⋅ 𝐴𝐶 = 𝑏𝑐
⇒ 𝑏2𝑐2 = 𝐴𝐿2𝑎⋅ 𝐴𝑆𝑎2 =


4𝑏𝑐


(𝑏 + 𝑐)2⋅ 𝑝(𝑝 − 𝑎) ⋅ 8𝑅𝑟 = 8 ⋅


4𝑏𝑐


(𝑏 + 𝑐)2⋅ 𝑝(𝑝 − 𝑎) ⋅


𝑎𝑏𝑐
4𝑆 ⋅



2𝑆
𝑎 + 𝑏 + 𝑐
⇒ (𝑏 + 𝑐)2 = 4𝑎(𝑏 + 𝑐 − 𝑎) ⇒ (𝑏 + 𝑐)2− 4𝑎(𝑏 + 𝑐) + 4𝑎2 = 0 ⇒ 2𝑎 = 𝑏 + 𝑐
Mặt khác ta có:


𝐼𝐿<sub>𝑎</sub>
𝐼𝐴 =


𝐵𝐿<sub>𝑎</sub>


𝑐 =


𝐶𝐿<sub>𝑎</sub>


𝑏 =


𝑎
𝑏 + 𝑐
Nên:


𝐼𝐺 ∥ 𝐵𝐶 ⇔𝐼𝐿𝑎
𝐼𝐴 =


𝐺𝑀<sub>𝑎</sub>
𝐺𝐴 =


1
2⇔


𝑎


𝑏 + 𝑐=


1


2⇔ 2𝑎 = 𝑏 + 𝑐
Nhưng đẳng thức cuối cùng là đúng do ta đã suy ra ở trên.


Từ đó ta có đpcm.


S<sub>a</sub>
O


L<sub>a</sub>
G


M<sub>a</sub>
I


A


B


</div>
<span class='text_page_counter'>(12)</span><div class='page_container' data-page=12>

<b>9. (IMO Shortlist) Giả sử M, N là các điểm nằm trong tam giác ABC sao cho 𝑴𝑨𝑩</b>̂ = 𝑵𝑨𝑪̂
<b>và 𝑴𝑩𝑨</b>̂ = 𝑵𝑩𝑪̂<b>. Chứng minh rằng: </b>


𝑨𝑴 ⋅ 𝑨𝑵
𝑨𝑩 ⋅ 𝑨𝑪 +


𝑩𝑴 ⋅ 𝑩𝑵
𝑩𝑪 ⋅ 𝑩𝑨 +



𝑪𝑴 ⋅ 𝑪𝑵
𝑪𝑨 ⋅ 𝑪𝑩 = 𝟏


<i>Lời giải:</i>


Xét phép vị tự quay 𝑆 có tâm 𝐵, góc quay (𝐵𝑀, 𝐵𝐴) (𝑚𝑜𝑑 𝜋) và tỉ số:
𝑘 = 𝐵𝐴


𝐵𝑀
Ta có: 𝑆: 𝑀 ↦ 𝐴, 𝐶 ↦ 𝐻 ∈ 𝐵𝑁. Khi đó:


(𝐻𝑁, 𝐻𝐶) ≡ (𝐴𝐵, 𝐴𝑀) ≡ (𝐴𝑁, 𝐴𝐶) (𝑚𝑜𝑑 𝜋)
Nên 𝐴, 𝑁, 𝐶, 𝐻 đồng viên. Theo định lý Ptolemy ta có:


𝐻𝐵 ⋅ 𝐴𝐶 = 𝐴𝐶 ⋅ (𝐵𝑁 + 𝑁𝐻) = 𝐴𝐶 ⋅ 𝐵𝑁 + 𝐴𝑁 ⋅ 𝐶𝐻 + 𝐴𝐻 ⋅ 𝐶𝑁
Lại theo tính chất của phép vị tự quay thì:


𝑘 = 𝐵𝐴
𝐵𝑀 =


𝐻𝐶
𝐴𝑀 =


𝐻𝐴
𝑀𝐶 =


𝐻𝐵


𝐵𝐶 ⇒ 𝐻𝐶 =



𝐴𝑀 ⋅ 𝐴𝐵


𝐵𝑀 ; 𝐻𝐵 =


𝐴𝐵 ⋅ 𝐵𝐶


𝐵𝑀 ; 𝐻𝐴 =


𝐴𝐵 ⋅ 𝑀𝐶
𝐵𝑀
Suy ra:


𝐴𝐵 ⋅ 𝐵𝐶


𝐵𝑀 ⋅ 𝐴𝐶 = 𝐴𝐶 ⋅ 𝐵𝑁 +


𝐴𝑀 ⋅ 𝐴𝐵


𝐵𝑀 ⋅ 𝐴𝑁 +


𝐴𝐵 ⋅ 𝑀𝐶
𝐵𝑀 ⋅ 𝐶𝑁
Hay:


𝐴𝑀 ⋅ 𝐴𝑁
𝐴𝐵 ⋅ 𝐴𝐶 +


𝐵𝑀 ⋅ 𝐵𝑁
𝐵𝐶 ⋅ 𝐵𝐴 +



𝐶𝑀 ⋅ 𝐶𝑁
𝐶𝐴 ⋅ 𝐶𝐵 = 1


H


N
A


B C


</div>
<span class='text_page_counter'>(13)</span><div class='page_container' data-page=13>

<b>10. (VMO 1997) Trong mặt phẳng, cho đường tròn </b>(𝑶, 𝑹) và điểm 𝑷 nằm trong đường
<b>tròn. Trong tất cả các tứ giác lồi 𝑨𝑩𝑪𝑫 nội tiếp trong đường trịn (𝑶) và có hai đường chéo </b>
𝑨𝑪 và 𝑩𝑫 vng góc và cắt nhau tại 𝑷, hãy tìm tứ giác có chu vi lớn nhất và tứ giác có chu
<b>vi nhỏ nhất. Tính các giá trị lớn nhất và nhỏ nhất này theo 𝑹 và 𝒅. </b>


<i>Lời giải: </i>


Gọi 𝐵𝐸 là đường kính của (𝑂) và đặt 𝑝 = 𝐴𝐵 + 𝐵𝐶 + 𝐶𝐷 + 𝐷𝐴. Ta có tam giác 𝐴𝐵𝐸 và tam
giác 𝑃𝐴𝐷 đồng dạng, tam giác 𝐶𝐵𝐸 và tam giác 𝑃𝐶𝐷 cũng đồng dạng nên:


𝐴𝐵 ⋅ 𝐴𝐷 = 2𝑅 ⋅ 𝑃𝐴; 𝐶𝐵 ⋅ 𝐶𝐷 = 2𝑅 ⋅ 𝐴𝐶
Suy ra:


𝐴𝐵 ⋅ 𝐴𝐷 + 𝐶𝐵 ⋅ 𝐶𝐷 = 2𝑅 ⋅ (𝑃𝐴 + 𝑃𝐶) = 2𝑅 ⋅ 𝐴𝐶
Tương tự:


𝐵𝐴 ⋅ 𝐵𝐶 + 𝐷𝐴 ⋅ 𝐷𝐶 = 2𝑅 ⋅ (𝑃𝐵 + 𝑃𝐷) = 2𝑅 ⋅ 𝐵𝐷
Mặt khác:



𝐴𝐵2<sub>+ 𝐶𝐷</sub>2 <sub>= 𝐴𝐵</sub>2<sub>+ 𝐴𝐸</sub>2 <sub>= 4𝑅</sub>2<sub>; 𝐴𝐷</sub>2<sub>+ 𝐵𝐶</sub>2 <sub>= 𝐶𝐸</sub>2<sub>+ 𝐵𝐶</sub>2 <sub>= 4𝑅</sub>2


Gọi 𝑀, 𝑁 là trung điểm của 𝐴𝐶, 𝐵𝐷, ta có:


𝐴𝐶2<sub>+ 𝐵𝐷</sub>2 <sub>= 4𝐴𝑀</sub>2<sub>+ 4𝐵𝑁</sub>2 <sub>= 4(𝑅</sub>2<sub>− 𝑂𝑀</sub>2<sub>) + 4(𝑅</sub>2<sub>− 𝑂𝑁</sub>2<sub>) = 8𝑅</sub>2<sub>− 4𝑑</sub>2


Ta lại có:


𝐴𝐶2 <sub>⋅ 𝐵𝐷</sub>2 <sub>= 16𝐴𝑀</sub>2<sub>⋅ 𝐵𝑁</sub>2 <sub>= 16(𝑅</sub>2<sub>− 𝑂𝑀</sub>2<sub>)(𝑅</sub>2<sub>− 𝑂𝑁</sub>2<sub>)</sub>


= 16(𝑅4− 𝑅2(𝑂𝑀2 <sub>+ 𝑂𝑁</sub>2<sub>) + 𝑂𝑀</sub>2 <sub>⋅ 𝑂𝑁</sub>2<sub>) = 16(𝑅</sub>4<sub>− 𝑅</sub>2<sub>𝑑</sub>2<sub>+ 𝑂𝑀</sub>2 <sub>⋅ 𝑂𝑁</sub>2<sub>) </sub>


Từ các đẳng thức trên ta có:


𝑝2 <sub>= 𝐴𝐵</sub>2<sub>+ 𝐶𝐷</sub>2<sub>+ 𝐴𝐷</sub>2 <sub>+ 𝐵𝐶</sub>2<sub>+ 2(𝐴𝐵 ⋅ 𝐴𝐷 + 𝐶𝐵 ⋅ 𝐶𝐷) + 2(𝐵𝐴 ⋅ 𝐵𝐶 + 𝐷𝐴 ⋅ 𝐷𝐶)</sub>


+ 2(𝐴𝐵 ⋅ 𝐶𝐷 + 𝐴𝐷 ⋅ 𝐵𝐶) = 8𝑅2+ 2𝐴𝐶 ⋅ 𝐵𝐷 + 4𝑅(𝐴𝐶 + 𝐵𝐷)
= 8𝑅2<sub>+ 2𝐴𝐶 ⋅ 𝐵𝐷 + 4𝑅√8𝑅</sub>2<sub>− 4𝑑</sub>2 <sub>+ 2𝐴𝐶 ⋅ 𝐵𝐷 </sub>


Thay 𝐴𝐶 ⋅ 𝐵𝐷 = 4√𝑅4<sub>− 𝑅</sub>2<sub>𝑑</sub>2<sub>+ 𝑂𝑀</sub>2<sub>⋅ 𝑂𝑁</sub>2<sub> vào biểu thức trên thì ta có: </sub>


𝑝2 = 8𝑅2+ 8√𝑅4<sub>− 𝑅</sub>2<sub>𝑑</sub>2<sub>+ 𝑂𝑀</sub>2<sub>⋅ 𝑂𝑁</sub>2<sub>+ 4𝑅√8𝑅</sub>2<sub>− 4𝑑</sub>2<sub>+ 8√𝑅</sub>4<sub>− 𝑅</sub>2<sub>𝑑</sub>2 <sub>+ 𝑂𝑀</sub>2<sub>⋅ 𝑂𝑁</sub>2


Vậy rõ ràng 𝑝 là một hàm tăng theo 𝑂𝑀2⋅ 𝑂𝑁2. Do đó:


N


M
P



E


C


O
D


</div>
<span class='text_page_counter'>(14)</span><div class='page_container' data-page=14>

1. 𝑝 nhỏ nhất khi và chỉ khi 𝑂𝑀2⋅ 𝑂𝑁2 = 0, tức là 𝐴𝐶 hoặc 𝐵𝐷 trở thành đường kính của (𝑂).
Giá trị nhỏ nhất của 𝑝 đạt được lúc này là:


𝑝𝑚𝑖𝑛 = 4 (𝑅2+ 𝑅√𝑅2− 𝑑2)
1
2


2. Theo bất đẳng thức AM-GM:


4𝑂𝑀2⋅ 𝑂𝑁2 ≤ (𝑂𝑀2+ 𝑂𝑁2)2 <sub>= 𝑑</sub>4


Đẳng thức xảy ra khi 𝐴𝐶 = 𝐵𝐷, khi đó 𝐴𝐶 và 𝐵𝐷 đối xứng nhau qua 𝑃, tức hợp với 𝑂𝑃 góc
bằng 45𝑜. Giá trị lớn nhất của 𝑝 đạt được lúc này là:


𝑝𝑚𝑎𝑥 = (16𝑅2− 4𝑑2+ 8𝑅√4𝑅2− 2𝑑2)
1
2


<b>11. (IMO SL 1997) Cho lục giác lồi </b>𝑨𝑩𝑪𝑫𝑬𝑭 có 𝑨𝑩 = 𝑩𝑪, 𝑪𝑫 = 𝑫𝑬, 𝑬𝑭 = 𝑭𝑨. Chứng
<b>minh rằng bất đẳng thức sau và cho biết đẳng thức xảy ra khi nào? </b>


𝑩𝑪
𝑩𝑬+



𝑫𝑬
𝑫𝑨+


𝑭𝑨
𝑭𝑪≥


𝟑
𝟐


<i>Lời giải: </i>


Đặt 𝐴𝐶 = 𝑥, 𝐶𝐸 = 𝑦, 𝐸𝐴 = 𝑧. Theo bất đẳng thức Ptolemy trong tứ giác 𝐴𝐶𝐸𝐹 ta có:
𝐴𝐶 ⋅ 𝐸𝐹 + 𝐶𝐸 ⋅ 𝐴𝐹 ≥ 𝐴𝐸. 𝐶𝐹


Nhưng 𝐹𝐸 = 𝐹𝐴 nên :


𝐹𝐴(𝑥 + 𝑦) ≥ 𝐹𝐶. 𝑧 hay𝐹𝐴
𝐹𝐶 ≥


𝑧
𝑥 + 𝑦
Lý luận tương tự ta cũng có :


𝐷𝐸
𝐷𝐴≥


𝑦
𝑧 + 𝑥 và



𝐵𝐶
𝐵𝐸 ≥


𝑥
𝑦 + 𝑧


Cộng các bất đẳng thức trên lại với nhau và kết hợp với bất đẳng thức Nesbit ta có :
𝐵𝐶


𝐵𝐸+
𝐷𝐸
𝐷𝐴+


𝐹𝐴
𝐹𝐶≥


𝑥
𝑦 + 𝑧+


𝑦
𝑧 + 𝑥+


𝑧
𝑥 + 𝑦≥


3
2


z



x


y


A B


D


C


</div>
<span class='text_page_counter'>(15)</span><div class='page_container' data-page=15>

Đẳng thức xảy ra khi và chỉ khi 𝑥 = 𝑦 = 𝑧 đồng thời 𝐴, 𝐵, 𝐶, 𝐷, 𝐸, 𝐹 đồng viên, tức 𝐴𝐵𝐶𝐷𝐸𝐹
phải là lục giác đều.


<b>12. (IMO 2001) Cho tam giác </b>𝑨𝑩𝑪 với trọng tâm 𝑮 và độ dài các cạnh 𝑩𝑪 = 𝒂, 𝑪𝑨 =
𝒃, 𝑨𝑩 = 𝒄. Tìm điểm 𝑷 trên mặt phẳng tam giác sao cho đại lượng 𝑨𝑮 ⋅ 𝑨𝑷 + 𝑩𝑷 ⋅ 𝑩𝑮 +
𝑪𝑷 ⋅ 𝑪𝑮 đạt giá trị nhỏ nhất và tìm giá trị nhỏ nhất đó theo a, b, c.


<i>Lời giải: </i>


Ta sử dụng nhất xét quen thuộc sau về tích vơ hướng của hai vector:
𝑎 ⋅ 𝑏⃗ = |𝑎 | ⋅ |𝑏⃗ | ⋅ cos(𝑎 , 𝑏⃗ ) ≤ |𝑎 | ⋅ |𝑏⃗ |
Theo nhận xét ta có:


𝐴𝑃 ⋅ 𝐴𝐺 + 𝐵𝑃 ⋅ 𝐵𝐺 + 𝐶𝑃 ⋅ 𝐶𝐺 ≥ 𝐴𝑃⃗⃗⃗⃗⃗ ⋅ 𝐴𝐺⃗⃗⃗⃗⃗ + 𝐵𝑃⃗⃗⃗⃗⃗ ⋅ 𝐵𝐺⃗⃗⃗⃗⃗ + 𝐶𝑃⃗⃗⃗⃗⃗ ⋅ 𝐶𝐺⃗⃗⃗⃗⃗
= 𝐴𝐺⃗⃗⃗⃗⃗ ⋅ (𝐴𝐺⃗⃗⃗⃗⃗ + 𝐺𝑃⃗⃗⃗⃗⃗ ) + 𝐵𝐺⃗⃗⃗⃗⃗ ⋅ (𝐵𝐺⃗⃗⃗⃗⃗ + 𝐺𝑃⃗⃗⃗⃗⃗ ) + 𝐶𝐺⃗⃗⃗⃗⃗ ⋅ (𝐶𝐺⃗⃗⃗⃗⃗ + 𝐺𝑃⃗⃗⃗⃗⃗ )


= 𝐺𝐴2+ 𝐺𝐵2+ 𝐺𝐶2+ 𝑃𝐺⃗⃗⃗⃗⃗ ⋅ (𝐴𝐺⃗⃗⃗⃗⃗ + 𝐵𝐺⃗⃗⃗⃗⃗ + 𝐶𝐺⃗⃗⃗⃗⃗ ) = 𝐺𝐴2+ 𝐺𝐵2+ 𝐺𝐶2
=𝑎


2 <sub>+ 𝑏</sub>2<sub>+ 𝑐</sub>2



3


Đẳng thức xảy ra khi và chỉ khi cos(𝐴𝑃⃗⃗⃗⃗⃗ ; 𝐴𝐺⃗⃗⃗⃗⃗ ) = cos(𝐵𝑃⃗⃗⃗⃗⃗ ; 𝐵𝐺⃗⃗⃗⃗⃗ ) = cos(𝐶𝑃⃗⃗⃗⃗⃗ ; 𝐶𝐺⃗⃗⃗⃗⃗ ) = 1 hay 𝑃 ≡ 𝐺.
<b>12*. Cho sáu điểm 𝑨, 𝑩, 𝑪, 𝑫, 𝑬, 𝑭 cùng thuộc đường tròn (𝑶, 𝑹) sao cho 𝑨𝑩 = 𝑪𝑫 = 𝑬𝑭 =</b>
𝑹. Gọi 𝑴, 𝑵, 𝑷 lần lượt là trung điểm 𝑩𝑪, 𝑫𝑬, 𝑭𝑨. Chứng minh rằng tam giác 𝑴𝑵𝑷 đều.


<i>Lời giải: </i>


Giả sử tam giác 𝑂𝐴𝐵 có hướng dương. Xét phép quay vector:
𝑄60°<sub>: 𝑂𝐴</sub><sub>⃗⃗⃗⃗⃗ ↦ 𝑂𝐵</sub><sub>⃗⃗⃗⃗⃗ ; 𝑂𝐶</sub><sub>⃗⃗⃗⃗⃗ ↦ 𝑂𝐷</sub><sub>⃗⃗⃗⃗⃗⃗ ; 𝑂𝐸</sub><sub>⃗⃗⃗⃗⃗ ↦ 𝑂𝐹</sub><sub>⃗⃗⃗⃗⃗ </sub>


Ta có:


P


N
M


E


C
B


O
A


</div>
<span class='text_page_counter'>(16)</span><div class='page_container' data-page=16>

𝑄(𝑀𝑁⃗⃗⃗⃗⃗⃗⃗ ) = 𝑄(𝑂𝑁⃗⃗⃗⃗⃗⃗ − 𝑂𝑀⃗⃗⃗⃗⃗⃗ ) = 𝑄 (1


2(𝑂𝐷⃗⃗⃗⃗⃗⃗ + 𝑂𝐸⃗⃗⃗⃗⃗ ) −


1


2(𝑂𝐵⃗⃗⃗⃗⃗ + 𝑂𝐶⃗⃗⃗⃗⃗ )) =
1


2𝑄(𝑂𝐷⃗⃗⃗⃗⃗⃗ + 𝑂𝐸⃗⃗⃗⃗⃗ − 𝑂𝐵⃗⃗⃗⃗⃗ − 𝑂𝐶⃗⃗⃗⃗⃗ )
=1


2(𝐶𝐷⃗⃗⃗⃗⃗ + 𝑂𝐹⃗⃗⃗⃗⃗ − 𝐴𝐵⃗⃗⃗⃗⃗ − 𝑂𝐷⃗⃗⃗⃗⃗⃗ ) =
1


2(𝐵𝐴⃗⃗⃗⃗⃗ + (𝐶𝐷⃗⃗⃗⃗⃗ + 𝐷𝑂⃗⃗⃗⃗⃗⃗ + 𝑂𝐹⃗⃗⃗⃗⃗ )) =
1


2(𝐵𝐴⃗⃗⃗⃗⃗ + 𝐶𝐹⃗⃗⃗⃗⃗ )
= 𝑀𝑃⃗⃗⃗⃗⃗⃗


Suy ra (𝑀𝑁⃗⃗⃗⃗⃗⃗⃗ , 𝑀𝑃⃗⃗⃗⃗⃗⃗ ) = 60° và 𝑀𝑁 = 𝑀𝑃 hay tam tam giác 𝑀𝑁𝑃 đều (đpcm).


<b>14. Cho hình thoi </b>𝑨𝑩𝑪𝑫 có 𝑫𝑨𝑩̂ = 𝟏𝟐𝟎°. 𝑴 ∈ 𝑨𝑫. 𝑩𝑴 cắt 𝑪𝑫 tại 𝑵. 𝑨𝑵 cắt 𝑪𝑴 tại 𝑷.
<b>Tính góc 𝑨𝑷𝑪</b>̂<b>. </b>


<i>Lời giải:</i>


Ta sẽ chứng minh 𝐴𝑃𝐶̂ = 60°, tức là 𝐴, 𝑃, 𝐶, 𝐷 đồng viên.


Dựng 𝑃′∈ 𝐶𝑀 sao cho 𝐴, 𝑃′, 𝐶, 𝐷 đồng viên. Đặt 𝑁′ ≡ 𝐴𝑃′∩ 𝐶𝐷, ta sẽ chứng minh 𝑃′≡ 𝑃
bằng cách chứng minh 𝐵, 𝑀, 𝑁′ thẳng hàng.


Điều đó tương đương với:



(𝐴𝐶, 𝐴𝐵, 𝐴𝑀, 𝐴𝑁′<sub>) = (𝐶𝐴, 𝐶𝐵, 𝐶𝑀, 𝐶𝑁</sub>′<sub>) </sub>


Nhưng do 𝐵𝐴, 𝐵𝐶 là tiếp tuyến của đường tròn (𝐴𝐶𝐷) nên:


(𝐴𝐶, 𝐴𝐵, 𝐴𝑀, 𝐴𝑁′<sub>) = 𝐴(𝐶𝐴𝐷𝑃</sub>′<sub>) = 𝐶(𝐶𝐴𝐷𝑃</sub>′<sub>) = (𝐶𝐵, 𝐶𝐴, 𝐶𝑁</sub>′<sub>, 𝐶𝑀) = (𝐶𝐴, 𝐶𝐵, 𝐶𝑀, 𝐶𝑁</sub>′<sub>) </sub>


Nên ta có đpcm.


<b>15. Cho tam giác 𝑨𝑩𝑪 nội tiếp trong đường tròn (𝑶). Gọi 𝑨</b><sub>𝟏</sub> ∈ 𝑩𝑪 là chân đường đối trung
<b>xuất phát từ </b>𝑨 của tam giác 𝑨𝑩𝑪. 𝑩<sub>𝒂</sub> ∈ 𝑨𝑪 và 𝑪<sub>𝒂</sub> ∈ 𝑨𝑩 sao cho 𝑨<sub>𝟏</sub>𝑩<sub>𝒂</sub>𝑨𝑪<sub>𝒂</sub><b> là hình bình </b>
<b>hành. </b>


<b>a. Chứng minh rằng 𝑩, 𝑪, 𝑩</b><sub>𝒂</sub>, 𝑪<sub>𝒂</sub><b> đồng viên. </b>


<b>b. Gọi 𝑶</b>𝒂<b> là tâm đường tròn (𝑩𝑪𝑩</b>𝒂𝑪𝒂). Chứng minh rằng 𝑨𝑶𝒂<b> đi qua trung điểm của 𝑶𝑳 </b>


<b>với 𝑳 là điểm Lemoine của tam giác 𝑨𝑩𝑪. </b>


<i>Lời giải:</i>


H


P


N


B A


C D



</div>
<span class='text_page_counter'>(17)</span><div class='page_container' data-page=17>

a. Ta có 𝐵𝐶𝐵<sub>𝑎</sub>𝐶<sub>𝑎</sub> là hình bình hành nên 𝐴𝐴<sub>1</sub> đi qua trung điểm của 𝐵<sub>𝑎</sub>𝐶<sub>𝑎</sub>, suy ra 𝐵, 𝐶, 𝐵<sub>𝑎</sub>, 𝐶<sub>𝑎</sub>
đồng viên.


b. Gọi 𝐼 là trung điểm của 𝑂𝐿. Xét phép vị tự:
ℋ<sub>𝐴</sub>𝑘=


𝐴𝐴1


𝐴𝐿<sub>: 𝐿 ↦ 𝐴</sub>


1, 𝐵 ↦ 𝐵′, 𝐶 ↦ 𝐶′, 𝑂 ↦ 𝑂′


Ta có 𝐴1 là điểm Lemoine của tam giác 𝐴𝐵′𝐶′ hơn nữa 𝐵𝐴̅̅̅̅̅̅̅ ∥ 𝐵1𝐶 ′𝐶′, 𝐴1𝐵𝑎 ∥ 𝐴𝐵′, 𝐴1𝐶𝑎 ∥ 𝐴𝐶′


nên (𝐵𝐶𝐵<sub>𝑎</sub>𝐶<sub>𝑎</sub>) chính là đường trịn Lemoine thứ nhất của tam giác 𝐴𝐵′𝐶′, từ đó vì 𝑂<sub>𝑎</sub> là tâm của
đường tròn này nên 𝑂<sub>𝑎</sub> là trung điểm của 𝑂′<sub>𝐴</sub>


1, mặt khác ℋ: 𝐿 ↦ 𝐴1, 𝑂 ↦ 𝑂′ nên ℋ: 𝐼 ↦ 𝑂𝑎.


Suy ra 𝐴, 𝐼, 𝑂𝑎 thẳng hàng (đpcm).


<b>16. Cho tam giác </b>𝑨𝑩𝑪 có 𝑨𝑨′, 𝑩𝑩′, 𝑪𝑪′ là các đường cao. Chứng minh rằng đường thẳng
<b>Euler của các tam giác 𝑨𝑩</b>′<sub>𝑪</sub>′<sub>, 𝑩𝑪</sub>′<sub>𝑨</sub>′<sub>, 𝑪𝑨</sub>′<sub>𝑩</sub>′<b><sub> đồng qui. </sub></b>


<i>Lời giải:</i>


O'


B' C'



I


O<sub>a</sub>
B<sub>a</sub>


C<sub>a</sub>


A<sub>1</sub>
L


S
O
A


B


</div>
<span class='text_page_counter'>(18)</span><div class='page_container' data-page=18>

Gọi 𝑂𝑎, 𝑂𝑏, 𝑂𝑐 là tâm đường tròn ngoại tiếp các tam giác 𝐴𝐵<i>′</i>𝐶<i>′</i>, 𝐵𝐶<i>′</i>𝐴<i>′</i>, 𝐶𝐴<i>′</i>𝐵<i>′</i> còn 𝐻𝑎, 𝐻𝑏, 𝐻𝑐 là


trực tâm của các tam giác ấy theo thứ tự.


Xét phép vị tự đối xứng 𝑆𝑎 có trục đối xứng là phân giác 𝑙𝑎 của góc 𝐵𝐴𝐶̂, tâm vị tự 𝐴 và tỉ số:


𝑘 =𝐴𝐵


<i>′</i>


𝐴𝐵 = cos 𝐴


Ta có 𝑆<sub>𝑎</sub>: 𝐵 ↦ 𝐵<i>′</i>, 𝐶 ↦ 𝐶<i>′</i>, 𝑂 ↦ 𝑂<sub>𝑎</sub>, 𝐻 ↦ 𝐻<sub>𝑎</sub> nên 𝑂<sub>𝑎</sub>𝐻<sub>𝑎</sub> và 𝑂𝐻 đối song với nhau trong góc 𝐵𝐴𝐶̂.


Gọi 𝐼 là tâm đường tròn nội tiếp tam giác 𝐴𝐵𝐶, ta biết rằng tích của hai phép đối xứng trục
𝐼𝐴, 𝐼𝐵 là phép quay tâm 𝐼, góc quay 2(𝐼𝐴, 𝐼𝐵) (𝑚𝑜𝑑 𝜋) nên:


(𝑂𝑎𝐻𝑎, 𝑂𝑏𝐻𝑏) ≡ 2(𝐼𝐴, 𝐼𝐵) ≡ 2((𝐼𝐴, 𝐴𝐵) + (𝐴𝐵, 𝐼𝐵)) ≡ (𝐶𝐴, 𝐴𝐵) + (𝐴𝐵, 𝐵𝐶)


≡ (𝐶𝐴, 𝐶𝐵) (𝑚𝑜𝑑 𝜋)


Mặt khác, 𝑂<sub>𝑐</sub>𝑂<sub>𝑎</sub> và 𝑂<sub>𝑐</sub>𝑂<sub>𝑏</sub> là đường trung bình của tam giác 𝐻𝐴𝐶 và tam giác 𝐻𝐴𝐵 nên:
(𝑂<sub>𝑐</sub>𝑂<sub>𝑎</sub>, 𝑂<sub>𝑐</sub>𝑂<sub>𝑏</sub>) ≡ (𝐶𝐴, 𝐶𝐵) ≡ (𝑂<sub>𝑎</sub>𝐻<sub>𝑎</sub>, 𝑂<sub>𝑏</sub>𝐻<sub>𝑏</sub>) (𝑚𝑜𝑑 𝜋)


Suy ra giao điểm 𝑆<sub>𝑐</sub> của 𝑂<sub>𝑎</sub>𝐻<sub>𝑎</sub> với 𝑂<sub>𝑏</sub>𝐻<sub>𝑏</sub> nằm trên đường tròn (𝑂<sub>𝑎</sub>𝑂<sub>𝑏</sub>𝑂<sub>𝑐</sub>), tức là đường tròn Euler
của tam giác 𝐴𝐵𝐶.


Lý luận tương tự thì ta có 𝑆𝑎 ≡ 𝑆𝑏 ≡ 𝑆𝑐 ≡ 𝑆 ∈ (Σ), hay 𝑂𝑎𝐻𝑎, 𝑂𝑏𝐻𝑏, 𝑂𝑐𝐻𝑐 đồng qui trên đường


trịn (Σ), trong đó (Σ) là đường tròn Euler của tam giác 𝐴𝐵𝐶 (đpcm).


<b>17. Cho tam giác 𝑨𝑩𝑪 nội tiếp trong đường trịn </b>(𝑶) có 𝑨′<sub>, 𝑩</sub>′<sub>, 𝑪′ là trung điểm các cung </sub>


𝑩𝑪, 𝑪𝑨, 𝑨𝑩 (không chứa 𝑨, 𝑩, 𝑪 tương ứng). Các cạnh của các tam giác 𝑨𝑩𝑪 và 𝑨′𝑩′𝑪′ cắt
<b>nhau tạo thành lục giác 𝑴𝑵𝑷𝑸𝑹𝑺 (𝑴, 𝑵 ∈ 𝑩𝑪; 𝑷, 𝑸 ∈ 𝑨𝑪; 𝑹, 𝑺 ∈ 𝑨𝑩). Chứng minh rằng: </b>


I


O<sub>b</sub>
H<sub>b</sub>


S


O<sub>c</sub>


H<sub>c</sub>
H<sub>a</sub>


O<sub>a</sub>


A'


B'


C'


H


O
A


</div>
<span class='text_page_counter'>(19)</span><div class='page_container' data-page=19>

𝟐


𝟑𝑺𝑨𝑩𝑪≤ 𝑺𝑴𝑵𝑷𝑸𝑹𝑺 ≤
𝟐


𝟑𝑺𝑨′𝑩′𝑪′


<i>Lời giải:</i>


Gọi 𝐼 là tâm đường tròn nội tiếp tam giác 𝐴𝐵𝐶.


Theo định lý Pascal ta có (𝑆, 𝐼, 𝑃), (𝑀, 𝐼, 𝑄), (𝑁, 𝐼, 𝑅) là các bộ ba điểm thẳng hàng. Hơn nữa
𝐶′<sub>𝐴 = 𝐶</sub>′<sub>𝐼, 𝐵</sub>′<sub>𝐴 = 𝐵</sub>′<sub>𝐼 nên 𝐵</sub>′<sub>𝐶</sub>′<sub> là đường trung trực của 𝐴𝐼, mặt khác 𝐴𝐼 phân giác góc 𝐵𝐴𝐶</sub><sub>̂</sub><sub> nên </sub>



𝐼𝑄𝐴𝑅 là hình thoi. Tương tự thì 𝐼𝑁𝐶𝑃 và 𝐼𝑀𝐵𝑆 cũng là các hình thoi. Suy ra 𝑃𝑆 ∥ 𝐵𝐶, 𝑁𝑅 ∥ 𝐴𝐶
và 𝑀𝑄 ∥ 𝐴𝐵. Từ đó:


𝐼𝑃
𝐼𝑆 =
𝐴𝐶
𝐴𝐵 =
𝑏
𝑐 ⇒
𝑃𝐼
𝑃𝑆=
𝑃𝑄
𝑃𝐴=
𝑏
𝑏 + 𝑐⇒
𝑆<sub>𝑃𝐼𝑄</sub>
𝑆<sub>𝐴𝑆𝑃</sub>=
𝑃𝐼
𝑃𝑆⋅
𝑃𝑄
𝑃𝐴=
𝑏2


(𝑏 + 𝑐)2


Tương tự thì:


𝑆<sub>𝑆𝐼𝑅</sub>
𝑆𝐴𝑆𝑃



= 𝑐


2


(𝑏 + 𝑐)2


Do đó:
𝑆<sub>𝐴𝑅𝐼𝑄</sub>


𝑆<sub>𝐴𝑆𝑃</sub> = 1 −
𝑏2


(𝑏 + 𝑐)2−


𝑐2


(𝑏 + 𝑐)2 =


2𝑏𝑐
(𝑏 + 𝑐)2 ⇒


𝑆<sub>𝐴𝑅𝑄</sub>
𝑆<sub>𝐴𝑆𝑃</sub> =


1
2 𝑆𝐴𝑅𝐼𝑄


𝑆<sub>𝐴𝑆𝑃</sub> =
1
2⋅



2𝑏𝑐
(𝑏 + 𝑐)2 =


𝑏𝑐
(𝑏 + 𝑐)2


</div>
<span class='text_page_counter'>(20)</span><div class='page_container' data-page=20>

𝐴𝐼
𝐴𝐵=


𝐼𝐴<sub>1</sub>
𝐴<sub>1</sub>𝐵=


𝐴𝐴<sub>1</sub>
𝐴𝐵 + 𝐴<sub>1</sub>𝐵
Nên suy ra:


𝐴𝑃
𝐴𝐶 =
𝐴𝑆
𝐴𝐵 =
𝑆𝑃
𝐵𝐶 =
𝐴𝐼
𝐴𝐴1
= 𝐴𝐵


𝐴𝐵 + 𝐴1𝐵


= 𝑐


𝑐 +<sub>𝑏 + 𝑐</sub>𝑐𝑎 =
𝑏 + 𝑐
𝑎 + 𝑏 + 𝑐
Do đó:
𝑆𝐴𝑆𝑃
𝑆<sub>𝐴𝐵𝐶</sub> =
𝐴𝑃
𝐴𝐶⋅
𝐴𝑆
𝐴𝐵 = (
𝑆𝑃
𝐵𝐶)
2
= (𝑏 + 𝑐)
2


(𝑎 + 𝑏 + 𝑐)2


Từ các điều trên ta có:
𝑆<sub>𝐴𝑅𝑄</sub>
𝑆<sub>𝐴𝐵𝐶</sub> =
𝑆<sub>𝐴𝑅𝑄</sub>
𝑆<sub>𝐴𝑆𝑃</sub> ⋅
𝑆𝐴𝑆𝑃
𝑆<sub>𝐴𝐵𝐶</sub> =
𝑏𝑐
(𝑏 + 𝑐)2⋅


(𝑏 + 𝑐)2



(𝑎 + 𝑏 + 𝑐)2 =


𝑏𝑐
(𝑎 + 𝑏 + 𝑐)2


Tương tự ta có:


𝑆<sub>𝐵𝑆𝑀</sub>
𝑆<sub>𝐴𝐵𝐶</sub> =


𝑎𝑐
(𝑎 + 𝑏 + 𝑐)2;


𝑆<sub>𝐶𝑃𝑁</sub>
𝑆<sub>𝐴𝐵𝐶</sub> =


𝑎𝑏
(𝑎 + 𝑏 + 𝑐)2


Suy ra:


𝑆<sub>𝑀𝑁𝑃𝑄𝑅𝑆</sub>
𝑆𝐴𝐵𝐶


= 1 −𝑎𝑏 + 𝑏𝑐 + 𝑐𝑎
(𝑎 + 𝑏 + 𝑐)2


Ta cần chứng minh:
𝑆<sub>𝑀𝑁𝑃𝑄𝑅𝑆</sub>



𝑆<sub>𝐴𝐵𝐶</sub> ≥
2


3⇔ 1 −


𝑎𝑏 + 𝑏𝑐 + 𝑐𝑎
(𝑎 + 𝑏 + 𝑐)2 ≥


2


3⇔ 3(𝑎𝑏 + 𝑏𝑐 + 𝑐𝑎) ≤ (𝑎 + 𝑏 + 𝑐)


2


Bất đẳng thức sau cùng hiển nhiên đúng nên ta có đpcm.
Bây giờ ta chứng minh chiều còn lại của bất đẳng thức.
Ta có:
𝑆<sub>𝐴</sub>′<sub>𝐵</sub>′<sub>𝐶</sub>′
𝑆<sub>𝐴𝐵𝐶</sub> =
𝐵′<sub>𝐶</sub>′
𝐵𝐶 ⋅
𝐶′<sub>𝐴</sub>′
𝐶𝐴 ⋅
𝐴′<sub>𝐵</sub>′
𝐴𝐵 =


𝑃<sub>𝐼,(𝑂)</sub>3


(𝐼𝐴 ⋅ 𝐼𝐵 ⋅ 𝐼𝐶)2 =



(2𝑅𝑟)3


( 𝑟
sin𝐴<sub>2</sub>


⋅ 𝑟
sin𝐵<sub>2</sub>


⋅ 𝑟
sin𝐶<sub>2</sub>


)


2


=8𝑅


3<sub>(sin</sub>𝐴


2 ⋅ sin
𝐵
2 ⋅ sin


𝐶
2)


2


𝑟3



Ta cần chứng minh:
2


3𝑆𝐴′𝐵′𝐶′ ≥ 𝑆𝑀𝑁𝑃𝑄𝑅𝑆 ⇔


16𝑅3<sub>(sin</sub>𝐴


2 ⋅ sin
𝐵
2 ⋅ sin


𝐶
2)


2


3𝑟3 ≥ 1 −


𝑎𝑏 + 𝑏𝑐 + 𝑐𝑎
(𝑎 + 𝑏 + 𝑐)2


⇔2
3⋅ 2 sin


2𝐴


2⋅ 2 sin


2𝐵



2⋅ 2 sin


2𝐶


2 ≥ (
𝑟
𝑅)


3


(1 −𝑎𝑏 + 𝑏𝑐 + 𝑐𝑎
(𝑎 + 𝑏 + 𝑐)2)


⇔2


3(1 − cos 𝐴)(1 − cos 𝐵)(1 − cos 𝐶) ≥ (cos 𝐴 + cos 𝐵 + cos 𝐶 − 1)


3<sub>(1 −</sub>𝑎𝑏 + 𝑏𝑐 + 𝑐𝑎


(𝑎 + 𝑏 + 𝑐)2)


⇔2


3∏ (1 −


𝑏2<sub>+ 𝑐</sub>2<sub>− 𝑎</sub>2


2𝑏𝑐 ) ≥ (1 −


𝑎𝑏 + 𝑏𝑐 + 𝑐𝑎


(𝑎 + 𝑏 + 𝑐)2) (∑


𝑏2 <sub>+ 𝑐</sub>2<sub>− 𝑎</sub>2


</div>
<span class='text_page_counter'>(21)</span><div class='page_container' data-page=21>

⇔(𝑏 + 𝑐 − 𝑎)


2<sub>(𝑐 + 𝑎 − 𝑏)</sub>2<sub>(𝑎 + 𝑏 − 𝑐)</sub>2


24𝑎3<sub>𝑏</sub>3<sub>𝑐</sub>3<sub>(𝑎 + 𝑏 + 𝑐)</sub>2 ⋅


⋅ (3(𝑎5+ 𝑏5 + 𝑐5) + 5𝑎𝑏𝑐(𝑎2<sub>+ 𝑏</sub>2 <sub>+ 𝑐</sub>2<sub>)</sub>


− 3(𝑎2<sub>𝑏</sub>2<sub>(𝑎 + 𝑏) + 𝑏</sub>2<sub>𝑐</sub>2<sub>(𝑏 + 𝑐) + 𝑐</sub>2<sub>𝑎</sub>2<sub>(𝑐 + 𝑎)) − 2𝑎𝑏𝑐(𝑎𝑏 + 𝑏𝑐 + 𝑐𝑎)) ≥ 0 </sub>


⇔ 3(𝑎5+ 𝑏5+ 𝑐5) + 5𝑎𝑏𝑐(𝑎2<sub>+ 𝑏</sub>2<sub>+ 𝑐</sub>2<sub>) − 3(𝑎</sub>2<sub>𝑏</sub>2<sub>(𝑎 + 𝑏) + 𝑏</sub>2<sub>𝑐</sub>2<sub>(𝑏 + 𝑐) + 𝑐</sub>2<sub>𝑎</sub>2<sub>(𝑐 + 𝑎))</sub>


− 2𝑎𝑏𝑐(𝑎𝑏 + 𝑏𝑐 + 𝑐𝑎) ≥ 0


Bất đẳng thức trên là tổng của hai bất đẳng thức sau:


2𝑎𝑏𝑐(𝑎2+ 𝑏2+ 𝑐2) ≥ 2𝑎𝑏𝑐(𝑎𝑏 + 𝑏𝑐 + 𝑐𝑎) ⇔ 𝑎𝑏𝑐((𝑎 − 𝑏)2<sub>+ (𝑏 − 𝑐)</sub>2<sub>+ (𝑐 − 𝑎)</sub>2<sub>) ≥ 0 </sub>


𝑎5 + 𝑏5+ 𝑐5+ 𝑎𝑏𝑐(𝑎2+ 𝑏2+ 𝑐2) ≥ 𝑎2<sub>𝑏</sub>2<sub>(𝑎 + 𝑏) + 𝑏</sub>2<sub>𝑐</sub>2<sub>(𝑏 + 𝑐) + 𝑐</sub>2<sub>𝑎</sub>2<sub>(𝑐 + 𝑎) </sub>


Trong đó bất đẳng thức thứ nhì đúng là vì từ bất đẳng thức Schur ta có:


𝑎3(𝑎 − 𝑏)(𝑎 − 𝑐) + 𝑏3<sub>(𝑏 − 𝑎)(𝑏 − 𝑐) + 𝑐</sub>3<sub>(𝑐 − 𝑎)(𝑐 − 𝑏) ≥ 0 </sub>


⇒ 𝑎5 + 𝑏5+ 𝑐5+ 𝑎𝑏𝑐(𝑎2+ 𝑏2+ 𝑐2) ≥ 𝑎𝑏(𝑎3<sub>+ 𝑏</sub>3<sub>) + 𝑏𝑐(𝑏</sub>3<sub>+ 𝑐</sub>3<sub>) + 𝑐𝑎(𝑐</sub>3<sub>+ 𝑎</sub>3<sub>)</sub>



≥ 𝑎2<sub>𝑏</sub>2<sub>(𝑎 + 𝑏) + 𝑏</sub>2<sub>𝑐</sub>2<sub>(𝑏 + 𝑐) + 𝑐</sub>2<sub>𝑎</sub>2<sub>(𝑐 + 𝑎) </sub>


(vì ta có 𝑎3<sub>+ 𝑏</sub>3 <sub>≥ 𝑎𝑏(𝑎 + 𝑏)). </sub>


Vậy ta có đpcm.


<b>18. Cho tam giác </b>𝑨𝑩𝑪, một đường thẳng 𝒅 bất kỳ cắt các cạnh 𝑩𝑪, 𝑪𝑨, 𝑨𝑩 theo thứ tự tại
𝑨𝟏, 𝑩𝟏, 𝑪𝟏<b>. 𝑷 là một điểm bất kỳ trên 𝒅. 𝑨</b>𝟐, 𝑩𝟐, 𝑪𝟐<b> là các điểm đối xứng với </b>𝑨𝟏, 𝑩𝟏, 𝑪𝟏<b> qua </b>


𝑷. Chứng minh rằng 𝑨𝑨<sub>𝟐</sub>, 𝑩𝑩<sub>𝟐</sub>, 𝑪𝑪<sub>𝟐</sub><b> đồng quy. </b>


<i>Lời giải: </i>


H


M


B<sub>2</sub>


C<sub>2</sub>


A2


C1


A


A1 B C


B1



</div>
<span class='text_page_counter'>(22)</span><div class='page_container' data-page=22>

Gọi 𝐵2, 𝐶2 là điểm đối xứng của 𝐵1, 𝐶1 qua 𝑃, 𝑀 ≡ 𝐵𝐵2∩ 𝐶𝐶2, 𝐻 ≡ 𝐴𝐶 ∩ 𝐵𝑀 và 𝐴′2 ≡ 𝐴𝑀 ∩ 𝑑,


ta sẽ chứng minh 𝑃 là trung điểm của 𝐴<sub>1</sub>𝐴<sub>2</sub>. Thật vậy, ta có:


(𝐴1𝐶1𝐵2𝐵1) = 𝐵(𝐴1𝐶1𝐵2𝐵1) = 𝐵(𝐶𝐴𝐻𝐵1) = 𝑀(𝐶𝐴𝐻𝐵1) = 𝑀(𝐶2𝐴2𝐵2𝐵1) = (𝐴2𝐶2𝐵1𝐵2)


Suy ra:


𝐴<sub>1</sub>𝐵<sub>2</sub>
̅̅̅̅̅̅̅
𝐴<sub>1</sub>𝐵<sub>1</sub>
̅̅̅̅̅̅̅:


𝐶<sub>1</sub>𝐵<sub>2</sub>
̅̅̅̅̅̅
𝐶<sub>1</sub>𝐵<sub>1</sub>
̅̅̅̅̅̅=


𝐴<sub>2</sub>𝐵<sub>1</sub>
̅̅̅̅̅̅̅
𝐴<sub>2</sub>𝐵<sub>2</sub>
̅̅̅̅̅̅̅:


𝐶<sub>2</sub>𝐵<sub>1</sub>
̅̅̅̅̅̅
𝐶<sub>2</sub>𝐵<sub>2</sub>
̅̅̅̅̅̅


Nhưng vì 𝐵2, 𝐶2 đối xứng với 𝐵1, 𝐶1 qua 𝑃 nên ta có 𝐶̅̅̅̅̅̅ = −𝐶1𝐵2 ̅̅̅̅̅̅2𝐵1 và 𝐶̅̅̅̅̅̅ = −𝐶1𝐵1 ̅̅̅̅̅̅2𝐵2. Suy ra:



𝐴1𝐵2


̅̅̅̅̅̅̅
𝐴<sub>1</sub>𝐵<sub>1</sub>
̅̅̅̅̅̅̅=


𝐴2𝐵1


̅̅̅̅̅̅̅
𝐴<sub>2</sub>𝐵<sub>2</sub>
̅̅̅̅̅̅̅ hay


𝐴1𝐵1


̅̅̅̅̅̅̅ + 𝐵̅̅̅̅̅̅̅1𝐵2


𝐴<sub>1</sub>𝐵<sub>1</sub>
̅̅̅̅̅̅̅ =


𝐴2𝐵2


̅̅̅̅̅̅̅ + 𝐵̅̅̅̅̅̅̅2𝐵1


𝐴<sub>2</sub>𝐵<sub>2</sub>
̅̅̅̅̅̅̅


Từ đó dẫn đến 𝐴̅̅̅̅̅̅̅ = 𝐵<sub>1</sub>𝐵<sub>1</sub> ̅̅̅̅̅̅̅<sub>2</sub>𝐴<sub>2</sub>, tức 𝑃 là trung điểm của 𝐴<sub>1</sub>𝐴<sub>2</sub> (vì 𝑃 đã là trung điểm 𝐵<sub>1</sub>𝐵<sub>2</sub>).
Kết thúc chứng minh.



<b>19. Tứ giác 𝑨𝑩𝑪𝑫 nội tiếp trong đường tròn (𝑶). Đường tròn ngoại tiếp tam giác </b>𝑶𝑨𝑩 và
<b>tam giác 𝑶𝑪𝑫 cắt nhau tại điểm </b>𝑲 ≠ 𝑶. Gọi 𝑳 là điểm nằm trong tứ giác 𝑨𝑩𝑪𝑫 sao cho
<b>tam giác 𝑨𝑫𝑳 đồng dạng với tam giác 𝑩𝑪𝑲. Chứng minh rằng tứ giác 𝑨𝑳𝑲𝑫 nội tiếp. </b>


<i>Lời giải: </i>


Ta có:


(𝐾𝐴, 𝐾𝐷) ≡ (𝐾𝐴, 𝐾𝑂) + (𝐾𝑂, 𝐾𝐷) ≡ (𝐵𝐴, 𝐵𝑂) + (𝐶𝑂, 𝐶𝐷) ≡ (𝐴𝑂, 𝐴𝐵) + (𝐷𝐶, 𝐷𝑂)
≡ (𝐾𝑂, 𝐾𝐵) + (𝐾𝐶, 𝐾𝑂) ≡ (𝐾𝐶, 𝐾𝐵) ≡ (𝐿𝐴, 𝐿𝐷) (𝑚𝑜𝑑 𝜋)


Suy ra 𝐴, 𝐿, 𝐾, 𝐷 đồng viên (đpcm).


L


K


O
A


B


</div>
<span class='text_page_counter'>(23)</span><div class='page_container' data-page=23>

<b>20. Cho đường trịn </b>(𝑶) có đường kính 𝑨𝑩. 𝑷 là một điểm bất kỳ trên đường tròn. 𝑲 là
<b>hình chiếu của 𝑷 trên 𝑨𝑩 và 𝑹 đối xứng với 𝑷 qua 𝑨𝑩. 𝑯 là một điểm bất kì trên 𝑨𝑩. 𝑹𝑯 </b>
<b>cắt (𝑶) lần nữa tại 𝑸. Gọi (𝑰, 𝒓) là đường tròn tiếp xúc với 𝑯𝑷, 𝑯𝑸 và (𝑶). Chứng minh hệ </b>
<b>thức: </b>


𝟏
𝒓=



𝟏
𝑯𝑲+


𝟏
𝑯𝑩


<i>Lời giải:</i>


Gọi 𝑇 là tiếp điểm của (𝐼) với 𝐻𝑄, 𝐽 là điểm thuộc 𝐼𝐻 sao cho 𝐻𝐽 = 𝐻𝑇, 𝑋 ≡ 𝑃𝑄 ∩ 𝐴𝐵 và 𝑊 ≡
𝐴𝑃 ∩ 𝐵𝑄, 𝐽′ <sub>≡ 𝐴𝑄 ∩ 𝐵𝑃, ta có 𝐴𝑄 ⊥ 𝐵𝑊 và 𝐵𝑃 ⊥ 𝐴𝑊 nên 𝐽′ là trực tâm của tam giác 𝑊𝐴𝐵, </sub>


suy ra 𝑊, 𝐽′, 𝐽, 𝐼, 𝐻 thẳng hàng. Hơn nữa:


𝐴𝐽<sub>̂ = 𝐽</sub>′<sub>𝐵</sub> <sub>̂ + 𝐴𝑃𝐵</sub>′<sub>𝐴𝑃</sub> <sub>̂ = 𝐽</sub><sub>̂ + 90° = 𝐽</sub>′<sub>𝐴𝑃</sub> <sub>̂ + 90° = 𝐽</sub>′<sub>𝐻𝑃</sub> <sub>̂ + 90° = 180° − 𝐻𝐼𝑇</sub>′<sub>𝐻𝑄</sub> <sub>̂</sub>


Hay:


𝐴𝐽<sub>̂ + 𝐻𝐼𝑇</sub>′<sub>𝐵</sub> <sub>̂ = 180° </sub>


Mặt khác:


X
J


W


K P


Q
R



T
H


B
O


S
I


</div>
<span class='text_page_counter'>(24)</span><div class='page_container' data-page=24>

tan 𝐴𝐽𝐵̂ = tan(𝐴𝐽𝐻̂ + 𝐵𝐽𝐻̂ ) = tan 𝐴𝐽𝐻̂ + tan 𝐵𝐽𝐻̂
1 − tan 𝐴𝐽𝐻̂ tan 𝐵𝐽𝐻̂ =


𝐴𝐻
𝐻𝐽 +


𝐵𝐻
𝐻𝐽
1 −𝐴𝐻<sub>𝐻𝐽 ⋅</sub>𝐵𝐻<sub>𝐻𝐽</sub>


=(𝐴𝐻 + 𝐵𝐻) ⋅ 𝐻𝐽
𝐻𝐽2 <sub>− 𝐻𝐴 ⋅ 𝐻𝐵</sub>


= 2𝑅 ⋅ 𝐻𝐽


𝐻𝑇2 <sub>− (𝑅</sub>2<sub>− 𝑂𝐻</sub>2<sub>)</sub> =


2𝑅 ⋅ 𝐻𝐽


𝐼𝐻2 <sub>− 𝑟</sub>2<sub>− 𝑅</sub>2<sub>+ 𝑂𝐻</sub>2 =



2𝑅 ⋅ 𝐻𝐽
𝐼𝑂2 <sub>− 𝑅</sub>2<sub>− 𝑟</sub>2


= 2𝑅 ⋅ 𝐻𝐽


(𝑅 − 𝑟)2<sub>− 𝑅</sub>2<sub>− 𝑟</sub>2 =


2𝑅 ⋅ 𝐻𝐽
−2𝑅𝑟 = −


𝐻𝐽
𝑟 = −


𝐻𝑇


𝑟 = − tan 𝐻𝐼𝑇̂
Suy ra 𝐴𝐽𝐵̂ + 𝐻𝐼𝑇̂ = 180°. Từ đó suy ra 𝐴𝐽𝐵̂ = 𝐴𝐽<sub>̂</sub>′<sub>𝐵</sub><sub> tức là 𝐽 ≡ 𝐽′. </sub>


Ta lại có tam giác 𝐻𝐾𝑅 và tam giác 𝐻𝑇𝐼 đồng dạng nên:
𝐵𝐻


𝐵𝐾 =
𝐻𝐽
𝑃𝐾=


𝐻𝑇
𝑅𝐾 =


𝐼𝐻


𝐻𝐾 =


𝑟
𝐻𝐾
Suy ra:


1
𝑟=


𝐵𝐾
𝐵𝐻 ⋅ 𝐻𝐾 =


𝐵𝐻 + 𝐻𝐾
𝐵𝐻 ⋅ 𝐻𝐾 =


1
𝐻𝐵+


1
𝐻𝐾
Đó chính là đpcm.


<b>21. Cho tam giác 𝑨𝑩𝑪 có phân giác 𝑨𝑫 và thỏa mãn các điều kiện 𝑨𝑪 + 𝑨𝑫 = 𝑩𝑪 và 𝑨𝑩 +</b>
𝑨𝑫 = 𝑪𝑫. Hãy tính các góc của tam giác 𝑨𝑩𝑪.


<i>Lời giải: </i>


Gọi 𝐻 là điểm đối xứng với 𝐵 qua 𝐴𝐷, ta có 𝐻 ∈ 𝐴𝐶. Mặt khác từ giả thiết 𝐴𝐶 + 𝐴𝐷 = 𝐵𝐶 và
𝐴𝐵 + 𝐴𝐷 = 𝐶𝐷 ta có 𝐻𝐶 = 𝐴𝐶 − 𝐴𝐻 = 𝐴𝐶 − 𝐴𝐵 = 𝐶𝐵 − 𝐶𝐷 = 𝐵𝐷 = 𝐻𝐷. Suy ra:



∠𝐴𝐶𝐵 =1


2∠𝐴𝐻𝐷 =
1


2∠𝐴𝐵𝐶


Lại gọi 𝐹 là điểm đối xứng với 𝐷 qua 𝐴𝐶. Ta có ∠𝐹𝐶𝐷 = 2∠𝐴𝐶𝐵 = ∠𝐴𝐵𝐶 nên:
𝐹𝐵 = 𝐹𝐶 = 𝐶𝐷 = 𝐴𝐵 + 𝐴𝐷


H
B


F


D


A


</div>
<span class='text_page_counter'>(25)</span><div class='page_container' data-page=25>

Suy ra 𝐵, 𝐴, 𝐹 thẳng hàng, hay ∠𝐹𝐴𝐶 = ∠𝐷𝐴𝐶 = ∠𝐷𝐴𝐵 = 60°.
Vậy ∠𝐵𝐴𝐶 = 120°, ∠𝐴𝐵𝐶 = 40° và ∠𝐴𝐶𝐵 = 20°.


<b>22. Về phía ngồi tam giác </b>𝑨𝑩𝑪 ta dựng các hình vng 𝑪𝑩𝑿𝒀, 𝑨𝑪𝒁𝑻, 𝑩𝑨𝑼𝑽. Về phía
<b>ngồi 𝑪𝑩𝑿𝒀 ta dựng tam giác </b>𝑲𝑿𝒀 vuông cân tại 𝑲. Chứng minh rằng 𝑨𝑲, 𝑿𝑻, 𝒀𝑼 đồng
<b>quy. </b>


<i>Lời giải: </i>


J <sub>L</sub>



I


K
U


V


Y
X


Z
T


A


</div>
<span class='text_page_counter'>(26)</span><div class='page_container' data-page=26>

Ký hiệu 𝐿 ≡ 𝐴𝑌 ∩ 𝐶𝑉, 𝐽 ≡ 𝐴𝑋 ∩ 𝐵𝑍. Ta có 𝑅(𝐶, 90°): 𝑍 ↦ 𝐴, 𝐵 ↦ 𝑌 nên ∠𝐴𝐾𝑍 = 90°, suy ra
𝐴, 𝐿, 𝐶, 𝑍, 𝑇 đồng viên. Mặt khác 𝑇𝐴 = 𝑇𝑍 nên 𝐿𝑇 là phân giác của ∠𝐴𝐿𝑍. Lý luận tương tự ta
cũng có 𝐿𝑋 là phân giác của ∠𝐵𝐿𝑌, từ đó suy ra 𝐿, 𝑇, 𝑋 thẳng hàng. Tương tự 𝐽, 𝑈, 𝑌 thằng hàng.
Hơn nữa, ta có ∠𝐶𝐽𝑋 = ∠𝐵𝐿𝑌 = 90° nên 𝑋, 𝑌, 𝐵, 𝐶, 𝐽, 𝐿 đồng viên. Đặt 𝐼 ≡ 𝑋𝑇 ∩ 𝑈𝑌, ta có:


(𝑋𝐴, 𝑋𝐼, 𝑋𝑌, 𝑋𝐾) = 𝑋(𝐽𝐿𝑌𝑋) = 𝑌(𝐽𝐿𝑌𝑋) = 𝑌(𝐿𝐽𝑋𝑌) = (𝑌𝐴, 𝑌𝐼, 𝑌𝑋, 𝑌𝐾)
Suy ra 𝐴, 𝐼, 𝐾 thẳng hàng, tức 𝐴𝐾, 𝑋𝑇, 𝑈𝑌 đồng quy (đpcm).


<b>24. Cho tam giác 𝑨𝑩𝑪 cân tại 𝑨. Đường thẳng 𝚫 đi qua 𝑨, song song với 𝑩𝑪. Các điểm 𝑷, 𝑸 </b>
<b>theo thứ tự thuộc các đường trung trực của 𝑨𝑩, 𝑨𝑪 sao cho 𝑷𝑸 ⊥ 𝑩𝑪 Các điểm 𝑴, 𝑵 thuộc </b>
∆ sao cho ∠𝑨𝑷𝑴 = ∠𝑨𝑸𝑵 = 𝟗𝟎°. Chứng minh rằng:


𝟏
𝑨𝑴+
𝟏


𝑨𝑵≤
𝟐
𝑨𝑩
<i>Lời giải: </i>


Đặt 𝑆 ≡ 𝑄𝑁 ∩ 𝑃𝑀, ta có ∠𝐴𝑄𝑆 = ∠𝐴𝑃𝑆 = 90° nên 𝐴, 𝑄, 𝑃, 𝑆 đồng viên.


Suy ra ∠𝐴𝑆𝑃 = ∠𝐴𝑄𝐻 = ∠𝐴𝑁𝑄, hơn nữa ∠𝐴𝑃𝑆 = ∠𝐴𝑄𝑁 = 90° nên tam giác 𝐴𝑃𝑆 và tam giác
𝐴𝑄𝑁 đồng dạng và tương tự tam giác 𝐴𝑄𝑆 cũng đồng dạng với tam giác 𝐴𝑃𝑀. Do đó:


𝐴𝑆
𝐴𝑀 =
𝐴𝑄
𝐴𝑃,
𝐴𝑆
𝐴𝑁=
𝐴𝑃
𝐴𝑄⇒
1
𝐴𝑀=
1
𝐴𝑆⋅
𝐴𝑄
𝐴𝑃;
1
𝐴𝑁=
1
𝐴𝑆⋅
𝐴𝑃
𝐴𝑄


Bất đẳng thức cần chứng minh tương đương với:


1
𝐴𝑀+
1
𝐴𝑁≤
2
𝐴𝐵 ⇔
𝐴𝐵
2
𝐴𝑆 ⋅
𝐴𝑄
𝐴𝑃+
𝐴𝐶
2
𝐴𝑆 ⋅
𝐴𝑃
𝐴𝑄≤ 1
Hay:


cos ∠𝑃𝐴𝐵 ⋅ cos ∠𝑆𝐴𝑄 + cos ∠𝑄𝐴𝐶 ⋅ cos ∠𝑆𝐴𝑃 ≤ 1


S


H N M


</div>
<span class='text_page_counter'>(27)</span><div class='page_container' data-page=27>

Mặt khác 𝑃𝑄 ⊥ 𝐵𝐶 nên tam giác 𝑂𝑃𝑄 cân tại 𝑂 và đồng dạng với tam giác 𝐴𝐵𝐶, cho ta:
∠𝑃𝐴𝐵 + ∠𝑆𝐴𝑄 = ∠𝑃𝐴𝐵 + ∠𝑃𝐴𝐻 = ∠𝐵𝐴𝐻 = 180° − ∠𝑂𝑃𝑄 = 180° − ∠𝑂𝑄𝑃


= 180° − ∠𝑂𝑄𝐻 = 180° − (∠𝑄𝐴𝐶 + ∠𝑆𝐴𝑃)



Suy ra: cos(∠𝑃𝐴𝐵 + ∠𝑆𝐴𝑄) + cos(∠𝑄𝐴𝐶 + ∠𝑆𝐴𝑃) = 0, nên bất đẳng thức tương đương với:
cos(∠𝑃𝐴𝐵 + ∠𝑆𝐴𝑄) + cos(∠𝑃𝐴𝐵 − ∠𝑆𝐴𝑄) + cos(∠𝑄𝐴𝐶 + ∠𝑆𝐴𝑃) + cos(∠𝑄𝐴𝐶 − ∠𝑆𝐴𝑃)


≤ 2


Hay: cos(∠𝑃𝐴𝐵 − ∠𝑆𝐴𝑄) + cos(∠𝑄𝐴𝐶 − ∠𝑆𝐴𝑃) ≤ 2
Bất đẳng thức sau cùng hiển nhiên đúng nên ta có đpcm.


Đẳng thức xảy ra khi và chỉ khi ∠𝑃𝐴𝐵 − ∠𝑆𝐴𝑄 = ∠𝑄𝐴𝐶 − ∠𝑆𝐴𝑃 = 0°, tức là 𝑃, 𝑄 nằm trên
đường tròn ngoại tiếp tam giác 𝐴𝐵𝐶.


<b>25. Tứ giác </b>𝑨𝑩𝑪𝑫 nội tiếp đường tròn (𝑶). Một đường thẳng ∆ bất kỳ trong mặt phẳng
<b>theo thứ tự cắt 𝑨𝑩, 𝑪𝑫, 𝑨𝑪, 𝑩𝑫, 𝑨𝑫, 𝑩𝑪 tại 𝑴, 𝑵, 𝑷, 𝑸, 𝑹, 𝑺 và cắt (𝑶) tại 𝑼, 𝑽. Chứng minh </b>
<b>rằng, nếu hai trong ba đoạn </b> 𝑴𝑵, 𝑷𝑸, 𝑹𝑺 có cùng trung điểm thì cả bốn đoạn
𝑴𝑵, 𝑷𝑸, 𝑹𝑺, 𝑼𝑽 có cùng trung điểm.


<i>Lời giải:</i>


Trước hết ta sẽ chứng minh nếu hai trong ba đoạn thẳng 𝑀𝑁, 𝑃𝑄, 𝑅𝑆 có cùng trung điểm thì cả
ba đoạn 𝑀𝑁, 𝑃𝑄, 𝑅𝑆 sẽ có cùng trung điểm.


Giả sử 𝑀𝑁, 𝑃𝑄 có cùng trung điểm. Ta sẽ chứng minh 𝑅𝑆 cũng có cùng trung điểm với hai đoạn
thẳng trên. Các trường hợp còn lại là tương tự. Thật vậy, đặt 𝐻 ≡ 𝐴𝐷 ∩ 𝐵𝐶, ta có:


(𝑀𝑅𝑃𝑆) = 𝐴(𝑀𝑅𝑃𝑆) = 𝐴(𝐵𝐻𝐶𝑆) = 𝐷(𝐵𝐻𝐶𝑆) = 𝐷(𝑄𝑅𝑁𝑆) = (𝑄𝑅𝑁𝑆) = (𝑁𝑆𝑄𝑅)


H


V


U


S
R


Q
P
A


B


D C


M


</div>
<span class='text_page_counter'>(28)</span><div class='page_container' data-page=28>

Suy ra:


𝑀𝑃
̅̅̅̅̅
𝑀𝑆
̅̅̅̅:


𝑅𝑃
̅̅̅̅
𝑅𝑆
̅̅̅̅ =


𝑁𝑄
̅̅̅̅
𝑁𝑅


̅̅̅̅:


𝑆𝑄
̅̅̅̅
𝑆𝑅
̅̅̅̅


Đơn giản 𝑅𝑆̅̅̅̅<sub> và với chú ý rằng 𝑀𝑁, 𝑃𝑄 có cùng trung điểm thì 𝑀𝑃</sub><sub>̅̅̅̅̅ = −𝑁𝑄</sub>̅̅̅̅<sub>, ta có: </sub>
𝑁𝑅


̅̅̅̅
𝑀𝑆
̅̅̅̅=


𝑅𝑃
̅̅̅̅
𝑆𝑄
̅̅̅̅ =


𝑁𝑅
̅̅̅̅ + 𝑅𝑃̅̅̅̅
𝑀𝑆


̅̅̅̅ + 𝑆𝑄̅̅̅̅ =
𝑁𝑃
̅̅̅̅
𝑀𝑄
̅̅̅̅̅ = 1


Từ đó suy ra 𝑁𝑅̅̅̅̅ = 𝑀𝑆̅̅̅̅ và 𝑅𝑃̅̅̅̅ = 𝑆𝑄̅̅̅̅, tức là 𝑀𝑁, 𝑃𝑄, 𝑅𝑆 có cùng trung điểm (đpcm).



Mặt khác ta sẽ chứng minh rằng nếu cả ba đoạn 𝑀𝑁, 𝑃𝑄, 𝑅𝑆 có cùng trung điểm thì 𝑈𝑉 cũng có
cùng trung điểm với chúng. Thật vậy:


(𝑀𝑈𝑃𝑉) = 𝐴(𝑀𝑈𝑃𝑉) = 𝐴(𝐵𝑈𝐶𝑉) = 𝐷(𝐵𝑈𝐶𝑉) = 𝐷(𝑄𝑈𝑁𝑉) = (𝑄𝑈𝑁𝑉) = (𝑁𝑉𝑄𝑈)
Áp dụng lại suy luận trên với ba đoạn thẳng 𝑀𝑁, 𝑃𝑄, 𝑈𝑉 và 𝑀𝑁, 𝑃𝑄 có cùng trung điểm ta suy
ra 𝑈𝑉 có cùng trung điểm với chúng (đpcm).


<b>26. Hai tứ giác </b>𝑨𝑩𝑷𝑸 và 𝑸𝑷𝑪𝑫 nội tiếp cùng hướng. Biết rằng tồn tại điểm 𝑬 ∈ 𝑷𝑸 sao
<b>cho ∠𝑬𝑨𝑷 = ∠𝑬𝑩𝑸 và ∠𝑬𝑪𝑸 = ∠𝑬𝑫𝑷. Chứng minh rằng 𝑨𝑩𝑪𝑫 nội tiếp. </b>


<i>Lời giải: </i>


Đặt 𝑋 ≡ 𝐴𝐸 ∩ (𝐴𝐵𝑃𝑄), 𝑌 ≡ 𝐵𝐸 ∩ (𝐴𝐵𝑃𝑄), 𝑍 ≡ 𝐶𝐸 ∩ (𝑄𝑃𝐶𝐷), 𝑇 ≡ 𝐷𝐸 ∩ (𝑄𝑃𝐶𝐷). Từ đề bài
ta có: ∠𝐸𝐴𝑃 = ∠𝐸𝐵𝑄 nên 𝑋𝑌𝑃𝑄 là hình thang cân, hơn nữa ∠𝐸𝐶𝑄 = ∠𝐸𝐷𝑃 nên 𝑍𝑇𝑄𝑃 cũng là
hình thang cân. Vậy phép đối xứng qua đường trung trực của 𝑃𝑄 biến 𝑍 ↦ 𝑇, 𝑋 ↦ 𝑌, suy ra
𝑋𝑌𝑍𝑇 cũng là hình thang cân, tức 𝑋, 𝑌, 𝑍, 𝑇 đồng viên.


C


D


B
A


X
T
P


Q



Y
Z


</div>
<span class='text_page_counter'>(29)</span><div class='page_container' data-page=29>

Mặt khác xét phép nghịch đảo ℐ có cực 𝐸, phương tích 𝑘 ≡ 𝐸𝑃̅̅̅̅ ⋅ 𝐸𝑄̅̅̅̅<sub>, ta có: </sub>
ℐ: 𝑃 ↦ 𝑄, 𝑋 ↦ 𝐴, 𝑌 ↦ 𝐵, 𝑍 ↦ 𝐶, 𝑇 ↦ 𝐷


Hơn nữa theo chứng minh trên thì 𝑋, 𝑌, 𝑍, 𝑇 đồng viên nên 𝐴, 𝐵, 𝐶, 𝐷 đồng viên hoặc thẳng hàng.
Tuy nhiên 𝐴, 𝐵, 𝐶, 𝐷 không thể thẳng hàng (tại sao?) nên chúng phải đồng viên và ta có đpcm.
<b>27. Cho 𝑻 là điểm Torricelli của tam giác 𝑨𝑩𝑪. 𝑨𝑻, 𝑩𝑻, 𝑪𝑻 cắt </b>𝑩𝑪, 𝑪𝑨, 𝑨𝑩 theo thứ tự tại
𝑨𝟎, 𝑩𝟎, 𝑪𝟎<b>, còn 𝑨</b>𝟏, 𝑩𝟏, 𝑪𝟏<b> cũng theo thứ tự ấy là các điểm đối xứng với </b>𝑻 qua 𝑩𝑪, 𝑪𝑨, 𝑨𝑩.


<b>Chứng minh rằng 𝑨</b><sub>𝟎</sub>𝑨<sub>𝟏</sub>, 𝑩<sub>𝟎</sub>𝑩<sub>𝟏</sub>, 𝑪<sub>𝟎</sub>𝑪<sub>𝟏</sub><b> đồng qui. </b>


<i>Lời giải: </i>


Gọi 𝐻 là điểm đẳng giác của 𝑇 đối với tam giác 𝐴𝐵𝐶. Ta sẽ chứng minh 𝐴0𝐴1, 𝐵0𝐵1, 𝐶0𝐶1 đồng


qui tại 𝐻. Đặc trưng, ta chỉ cần chứng minh 𝐴1, 𝐴0, 𝐻 thẳng hàng.


Thật vậy, gọi 𝐴′ là điểm đối xứng với 𝐴 qua 𝐵𝐶, ta thấy 𝐴<sub>1</sub> là điểm Torricelli của tam giác 𝐴′𝐵𝐶
nên 𝐴′<sub>, 𝐴</sub>


1, 𝐴0 thẳng hàng. 𝐻, 𝐴′, 𝐴1 thẳng hàng khi và chỉ khi:


(𝐵𝐻, 𝐵𝐶, 𝐵𝐴<sub>1</sub>, 𝐵𝐴′<sub>) = (𝐶𝐻, 𝐶𝐵, 𝐶𝐴</sub>
1𝐶𝐴′)


⇔sin(𝐵𝐻⃗⃗⃗⃗⃗⃗ , 𝐵𝐴⃗⃗⃗⃗⃗⃗⃗⃗ )1
sin (𝐵𝐻⃗⃗⃗⃗⃗⃗ , 𝐵𝐴′⃗⃗⃗⃗⃗⃗⃗ ):



sin(𝐵𝐶⃗⃗⃗⃗⃗ , 𝐵𝐴⃗⃗⃗⃗⃗⃗⃗⃗ )<sub>1</sub>
sin (𝐵𝐶⃗⃗⃗⃗⃗ , 𝐵𝐴′⃗⃗⃗⃗⃗⃗⃗ )=


sin(𝐶𝐻⃗⃗⃗⃗⃗ , 𝐶𝐴⃗⃗⃗⃗⃗⃗⃗ )<sub>1</sub>
sin (𝐶𝐻⃗⃗⃗⃗⃗ , 𝐶𝐴′⃗⃗⃗⃗⃗⃗ ):


sin(𝐶𝐵⃗⃗⃗⃗⃗ , 𝐶𝐴⃗⃗⃗⃗⃗⃗⃗ )<sub>1</sub>
sin(𝐶𝐵⃗⃗⃗⃗⃗ , 𝐶𝐴′)


Nhưng (𝐵𝐻⃗⃗⃗⃗⃗⃗ , 𝐵𝐴⃗⃗⃗⃗⃗⃗⃗⃗ ) ≡ (𝐵𝐻<sub>1</sub> ⃗⃗⃗⃗⃗⃗ , 𝐵𝐶⃗⃗⃗⃗⃗ ) + (𝐵𝐶⃗⃗⃗⃗⃗ , 𝐵𝐴⃗⃗⃗⃗⃗⃗⃗⃗ ) ≡ (𝐵𝐴<sub>1</sub> ⃗⃗⃗⃗⃗ , 𝐵𝐵⃗⃗⃗⃗⃗⃗⃗⃗ ) + (𝐵𝐵<sub>0</sub> ⃗⃗⃗⃗⃗⃗⃗⃗ , 𝐵𝐶<sub>0</sub> ⃗⃗⃗⃗⃗ ) = (𝐵𝐴⃗⃗⃗⃗⃗ , 𝐵𝐶⃗⃗⃗⃗⃗ ) và tương
tự thì (𝐶𝐵⃗⃗⃗⃗⃗ , 𝐶𝐴⃗⃗⃗⃗⃗⃗⃗ ) ≡ (𝐶𝐴1 ⃗⃗⃗⃗⃗ , 𝐶𝐵⃗⃗⃗⃗⃗ ). Do vậy, hệ thức trên tương đương với:


A<sub>1</sub>


A'


H
C<sub>0</sub>


B<sub>0</sub>


A<sub>0</sub>
T


A


</div>
<span class='text_page_counter'>(30)</span><div class='page_container' data-page=30>

(sin(𝐵𝐴⃗⃗⃗⃗⃗ , 𝐵𝐶⃗⃗⃗⃗⃗ )
sin(𝐶𝐴⃗⃗⃗⃗⃗ , 𝐶𝐵⃗⃗⃗⃗⃗ ))



2


=sin (𝐵𝐻⃗⃗⃗⃗⃗⃗ , 𝐵𝐴′⃗⃗⃗⃗⃗⃗⃗ ) ⋅ sin(𝐵𝐶⃗⃗⃗⃗⃗ , 𝐵𝐴⃗⃗⃗⃗⃗⃗⃗⃗ )1
sin (𝐶𝐻⃗⃗⃗⃗⃗ , 𝐶𝐴′⃗⃗⃗⃗⃗⃗ ) ⋅ sin(𝐶𝐵⃗⃗⃗⃗⃗ , 𝐶𝐴⃗⃗⃗⃗⃗⃗⃗ )<sub>1</sub>


⇔𝐴𝐶


2


𝐴𝐵2 =


sin(𝐵𝑇⃗⃗⃗⃗⃗ , 𝐵𝐶⃗⃗⃗⃗⃗ )
sin (𝐶𝑇⃗⃗⃗⃗⃗ , 𝐶𝐵′⃗⃗⃗⃗⃗⃗ )⋅


sin (𝐵𝐻⃗⃗⃗⃗⃗⃗ , 𝐵𝐴′⃗⃗⃗⃗⃗⃗⃗ )
sin (𝐶𝐻⃗⃗⃗⃗⃗ , 𝐶𝐴′⃗⃗⃗⃗⃗⃗ )
Mặt khác áp dụng liên tiếp định lý sin thì ta có:


sin (𝐵𝐻⃗⃗⃗⃗⃗⃗ , 𝐵𝐴′⃗⃗⃗⃗⃗⃗⃗ )
sin (𝐶𝐻⃗⃗⃗⃗⃗ , 𝐶𝐴′⃗⃗⃗⃗⃗⃗ )=


𝑆<sub>𝐵𝐻𝐴</sub>′
̅̅̅̅̅̅̅
𝑆̅̅̅̅̅̅̅𝐶𝐻𝐴′ ⋅
𝐶𝐻
𝐵𝐻⋅
𝐶𝐴′
𝐵𝐴′ =


𝐴<sub>0</sub>𝐵


̅̅̅̅̅
𝐴0𝐶


̅̅̅̅̅⋅
𝐴𝐶
𝐴𝐵⋅


sin ∠𝐻𝐵𝐶
sin ∠𝐻𝐶𝐵 =


𝐴<sub>0</sub>𝐵
̅̅̅̅̅
𝐴0𝐶


̅̅̅̅̅⋅
𝐴𝐶
𝐴𝐵⋅


sin ∠𝐴𝐵𝑇
sin ∠𝐴𝐶𝑇


=𝐴̅̅̅̅̅0𝐵
𝐴0𝐶


̅̅̅̅̅⋅
𝐴𝐶
𝐴𝐵⋅
𝐴𝑇
sin ∠𝐴𝐶𝑇
𝐴𝑇


sin ∠𝐴𝐵𝑇


=𝐴̅̅̅̅̅0𝐵
𝐴0𝐶


̅̅̅̅̅⋅
𝐴𝐶
𝐴𝐵⋅
𝐴𝐶
sin ∠𝐴𝑇𝐶
𝐴𝐵
sin ∠𝐴𝐵𝑇


= 𝐴̅̅̅̅̅0𝐵
𝐴0𝐶


̅̅̅̅̅ ⋅
𝐴𝐶2


𝐴𝐵2


Hơn nữa thì:


sin(𝐵𝑇⃗⃗⃗⃗⃗ , 𝐵𝐶⃗⃗⃗⃗⃗ )
sin (𝐶𝑇⃗⃗⃗⃗⃗ , 𝐶𝐵′⃗⃗⃗⃗⃗⃗ )=


𝑆<sub>𝑇𝐴</sub><sub>0</sub><sub>𝐵</sub>
̅̅̅̅̅̅̅
𝑆<sub>𝑇𝐴</sub><sub>0</sub><sub>𝐶</sub>
̅̅̅̅̅̅̅=



𝐴<sub>0</sub>𝐶
̅̅̅̅̅
𝐴<sub>0</sub>𝐵
̅̅̅̅̅
Nên suy ra:


sin (𝐵𝐻⃗⃗⃗⃗⃗⃗ , 𝐵𝐴′⃗⃗⃗⃗⃗⃗⃗ )
sin (𝐶𝐻⃗⃗⃗⃗⃗ , 𝐶𝐴′⃗⃗⃗⃗⃗⃗ )⋅


sin(𝐵𝑇⃗⃗⃗⃗⃗ , 𝐵𝐶⃗⃗⃗⃗⃗ )
sin (𝐶𝑇⃗⃗⃗⃗⃗ , 𝐶𝐵′⃗⃗⃗⃗⃗⃗ )=


𝐴0𝐵


̅̅̅̅̅
𝐴<sub>0</sub>𝐶
̅̅̅̅̅ ⋅


𝐴𝐶2
𝐴𝐵2⋅


𝐴0𝐶


̅̅̅̅̅
𝐴<sub>0</sub>𝐵
̅̅̅̅̅ =


𝐴𝐶2
𝐴𝐵2



Do vậy, đẳng thức cần chứng minh là đúng và ta có đpcm.


<b>28. Cho tam giác 𝑨𝑩𝑪 có 𝑴 là trung điểm của cạnh 𝑨𝑩. 𝑪𝑬 là phân giác của góc ∠𝑨𝑪𝑩. 𝑫 </b>
<b>thuộc tia đối của tia 𝑪𝑨 sao cho 𝑪𝑫 = 𝑪𝑩. Đặt 𝑲 ≡ 𝑫𝑴 ∩ 𝑪𝑬. Chứng minh rằng: </b>


</div>
<span class='text_page_counter'>(31)</span><div class='page_container' data-page=31>

Lấy 𝐽 ∈ 𝐶𝐴 sao cho 𝐶𝐽 = 𝐶𝐵 = 𝐶𝐷, ta có 𝐵𝐽 ⊥ 𝐵𝐷 nên 𝐵𝐽 ⊥ 𝐶𝐸. Lại đặt 𝐹 ≡ 𝐶𝐸 ∩ (𝑂), 𝐻 ≡
𝐹𝐽 ∩ (𝑂). Ta có:


(𝐻𝐵, 𝐻𝐹) ≡ (𝐶𝐵, 𝐶𝐹) ≡ (𝐷𝐵, 𝐷𝐽) (𝑚𝑜𝑑 𝜋)
Suy ra 𝐵, 𝐷, 𝐽, 𝐻 đồng viên, ta có 𝐶𝐵 = 𝐶𝐷 = 𝐶𝐽 = 𝐶𝐻. Từ đó:


(𝐴𝐵, 𝐴𝐶) ≡ (𝐹𝐵, 𝐹𝐶) ≡ (𝐹𝐶, 𝐹𝐻) ≡ (𝐵𝐶, 𝐵𝐻) (𝑚𝑜𝑑 𝜋)
Mặt khác vì 𝐾𝐸 ∥ 𝐵𝐷 và theo tính chất đường phân giác thì:


𝐾𝐷
𝐷𝑀=


𝐸𝐵
𝐵𝑀=


2𝐸𝐵
𝐸𝐵 + 𝐸𝐴=


2𝐶𝐵
𝐶𝐵 + 𝐶𝐴=


𝐽𝐵
𝐷𝐴
Nên 𝐾𝐽 ∥ 𝑀𝐴, từ đó:



𝐿𝐾
̅̅̅̅
𝐿𝐵
̅̅̅̅ =


𝐿𝐽̅
𝐿𝐴
̅̅̅̅⇒


𝐿𝐾
̅̅̅̅ ⋅ 𝐿𝐻̅̅̅̅
𝐿𝐵


̅̅̅̅ ⋅ 𝐿𝐻̅̅̅̅ =


𝐿𝐽̅ ⋅ 𝐿𝐶̅̅̅̅
𝐿𝐴
̅̅̅̅ ⋅ 𝐿𝐶̅̅̅̅


Mà 𝐿𝐾̅̅̅̅ ⋅ 𝐿𝐻̅̅̅̅ = 𝐿𝐽̅ ⋅ 𝐿𝐶̅̅̅̅ nên 𝐿𝐵̅̅̅̅ ⋅ 𝐿𝐻̅̅̅̅ = 𝐿𝐴̅̅̅̅ ⋅ 𝐿𝐶̅̅̅̅, suy ra 𝐶, 𝐻, 𝐽, 𝐾 đồng viên. Vậy nên:
(𝐻𝐾, 𝐻𝐶) ≡ (𝐽𝐾, 𝐽𝐶) ≡ (𝐴𝐵, 𝐴𝐶) ≡ (𝐻𝐵, 𝐻𝐶) (𝑚𝑜𝑑 𝜋)
Suy ra 𝐻, 𝐾, 𝐵 thẳng hàng. Do đó:


(𝐴𝐵, 𝐴𝐶) ≡ (𝐵𝐶, 𝐵𝐾) (𝑚𝑜𝑑 𝜋)
Tức là ∠𝐾𝐵𝐶 = ∠𝐵𝐴𝐶 (đpcm).


<b>29. Cho đường tròn </b>(𝑶) và điểm 𝑨 nằm trong (𝑶). Các điểm 𝑩, 𝑪 nằm trên (𝑶) sao cho
<b>tam giác </b>𝑨𝑩𝑪 đều. 𝑫 ∈ (𝑶) sao cho 𝑪𝑫 = 𝑪𝑩 (𝑫 ≠ 𝑩). 𝑫𝑨 ∩ (𝑶) = 𝑬. Chứng minh rằng
𝑬𝑨 = 𝑬𝑶.



<i>Lời giải:</i>


Ta có:


∠𝐸𝐴𝐵 = 180° − ∠𝐵𝐴𝐶 − ∠𝐶𝐴𝐷 = 180° − 60° − ∠𝐶𝐷𝐴 = ∠𝐸𝐵𝐶 − ∠𝐴𝐵𝐶 = ∠𝐸𝐵𝐴
Nên tam giác 𝐸𝐴𝐵 cân tại 𝐸, tức là 𝐸𝐴 = 𝐸𝐵.


Mặt khác ∠𝐵𝑂𝐶 = ∠𝐷𝑂𝐶 = 2∠𝐶𝐸𝐷 = ∠𝐴𝐸𝐵.


Tam giác 𝐵𝐸𝐴 và tam giác 𝐵𝑂𝐶 cân tại 𝐸, 𝑂, lại có ∠𝐵𝐸𝐴 = ∠𝐵𝑂𝐶 và 𝐵𝐴 = 𝐵𝐶 nên chúng
bằng nhau, từ đó suy ra 𝐸𝐴 = 𝑂𝐶 = 𝐸𝑂 (đpcm).


E


D
A


O


</div>
<span class='text_page_counter'>(32)</span><div class='page_container' data-page=32>

<b>30. Cho hình thang 𝑨𝑩𝑪𝑫 (𝑨𝑩 ∥ 𝑪𝑫). 𝑴 là một điểm trên cạnh 𝑨𝑫 và 𝑵 nằm trong hình </b>
<b>thang sao cho: </b>∠𝑵𝑩𝑪 = ∠𝑴𝑩𝑨 và ∠𝑵𝑪𝑩 = ∠𝑴𝑪𝑫. Chứng minh rằng đỉnh 𝑷 của hình
<b>bình hành 𝑴𝑩𝑵𝑷 thuộc 𝑪𝑫. </b>


<i>Lời giải:</i>


Đặt 𝐾 ≡ 𝑁𝐵 ∩ 𝐶𝐷. Qua 𝑀 vẽ đường thẳng song song với 𝑀𝐵, cắt 𝐶𝐷 tại 𝑄. Ta có:
∠𝑀𝑄𝐶 = ∠𝐴𝐵𝑀 = ∠𝐶𝐵𝑁 và ∠𝐵𝐶𝑁 = ∠𝑄𝑀𝐶


Nên tam giác 𝐶𝑀𝑄 và tam giác 𝐶𝑁𝐵 đồng dạng, hơn nữa cũng từ đó thì tam giác 𝐶𝐵𝐾 và tam


giác 𝐶𝑄𝐵 cũng đồng dạng. Từ các tỉ số bằng nhau ta có:


𝑄𝑀
𝐵𝑁 =


𝐶𝑄
𝐶𝐵=


𝑄𝐵
𝐵𝐾
Suy ra:


𝑄𝑀
𝑄𝐵 =


𝐵𝑁
𝐵𝐾 =


𝑄𝑃
𝑄𝐾
Từ đó 𝑀𝑃 ∥ 𝐵𝐾 nên 𝐵𝑀𝑃𝑁 là hình bình hành và ta có đpcm.
Ghi chú: điều kiện 𝑀 ∈ 𝐴𝐷 là khơng cần thiết.


<b>31. Cho đường trịn (𝑶) và điểm 𝑨 nằm ngoài(𝑶). 𝑨𝑩, 𝑨𝑪 là các tiếp tuyến kẻ từ 𝑨 tới(𝑶). </b>
𝑫 là giao điểm của 𝑨𝑶 và(𝑶). 𝑿 là hình chiếu của 𝑩 trên 𝑪𝑫. 𝒀 là trung điểm của 𝑩𝑿. 𝒁 là
<b>giao điểm thứ hai của 𝑫𝒀 và(𝑶). Chứng minh rằng ∠𝑨𝒁𝑪 = 𝟗𝟎°. </b>


<i>Lời giải: </i>


K


Q


P


N


A B


D C


</div>
<span class='text_page_counter'>(33)</span><div class='page_container' data-page=33>

Đặt 𝑍 ≡ 𝐷𝑌 ∩ (𝑂) và 𝐶𝑇 là đường kính của (𝑂). Ta có 𝐷𝑇 ⊥ 𝐷𝐶 nên 𝐷𝑇 ∥ 𝐵𝑋. Hơn nữa 𝑌 là
trung điểm 𝐵𝑋 nên (𝐷𝑇, 𝐷𝑌, 𝐷𝑋, 𝐷𝐵) = −1. Từ đó:


𝐷(𝑇𝑍𝐵𝐶) = (𝐷𝑇, 𝐷𝑌, 𝐷𝑋, 𝐷𝐵) = −1


Suy ra 𝑇𝐵𝑍𝐶 là tứ giác điều hòa. Mặt khác 𝐴 là cực của 𝐵𝐶 đối với đường tròn (𝑂) nên 𝐴, 𝑍, 𝑇
thẳng hàng. Vậy nên ∠𝐴𝑍𝐶 = 90° (đpcm).


<b>32. Cho tứ giác lồi 𝑨𝑩𝑪𝑫. Gọi 𝒂, 𝒃, 𝒄, 𝒅 là phân giác ngoài các góc ∠𝑫𝑨𝑩, ∠𝑨𝑩𝑪, ∠𝑩𝑪𝑫 và </b>
∠𝑪𝑫𝑨. 𝑲, 𝑳, 𝑴, 𝑵 lần lượt là giao điểm của 𝒂, 𝒃; 𝒃, 𝒄; 𝒄, 𝒅; 𝒅, 𝒂. Giả sử các đường thẳng qua
𝑲, 𝑳, 𝑴, 𝑵 và tương ứng vng góc với 𝑨𝑩, 𝑩𝑪, 𝑪𝑫, 𝑫𝑨 đồng qui. Chứng minh rằng 𝑨𝑩𝑪𝑫
<b>là tứ giác nội tiếp. </b>


<i>Lời giải: </i>


Gọi 𝐼 là điểm đồng qui của bốn đường thẳng qua 𝐾, 𝐿, 𝑀, 𝑁 và tương ứng vuông góc với
𝐴𝐵, 𝐵𝐶, 𝐶𝐷, 𝐷𝐴. Ta có 𝑀𝑁 là đường phân giác ngoài của ∠𝐴𝐷𝐶 nên ∠𝑁𝐷𝐴 = ∠𝑀𝐷𝐶, hơn nữa


Y X



Z
T


D


C
B


A


O


T Y


X
Z


I
N


M


L
K


O
D


C
B



</div>
<span class='text_page_counter'>(34)</span><div class='page_container' data-page=34>

∠𝑁𝐷𝐴 + ∠𝐼𝑁𝑀 = ∠𝑀𝐷𝐶 + ∠𝐼𝑀𝑁 = 90° nên ∠𝐼𝑁𝑀 = ∠𝐼𝑀𝑁, suy ra 𝐼𝑀 = 𝐼𝑁. Lý luận tương
tự ta có 𝐼𝐾 = 𝐼𝐿 = 𝐼𝑀 = 𝐼𝑁, do đó 𝐾, 𝐿, 𝑀, 𝑁 thuộc đường trịn tâm 𝐼. Từ đó:


∠𝐵𝐴𝐷 + ∠𝐵𝐶𝐷 = 180° − 2∠


<b>33. Tam giác 𝑨𝑩𝑪 nội tiếp trong đường tròn </b>(𝑶) có 𝑩𝑪 > 𝑪𝑨 > 𝑨𝑩. 𝑰 là tâm đường tròn
<b>nội tiếp tam giác </b>𝑨𝑩𝑪. 𝑨𝑰 ∩ (𝑶) ≡ 𝑲, 𝑶𝑲 ∩ 𝑩𝑪 = 𝑴 và 𝑵 là điểm đối xứng của 𝑰 qua 𝑴.
𝑲𝑵 ∩ (𝑶) ≡ 𝑳. Chứng minh rằng 𝑳𝑩 = 𝑳𝑨 + 𝑳𝑪.


<i>Lời giải:</i>


Xét phép nghịch đảo ℐ có cực 𝐾, phương tích 𝑘 ≔ 𝐾𝐵2 <sub>= 𝐾𝐶</sub>2<sub>. Ta có: </sub>


ℐ: 𝐵 ↦ 𝐵, 𝐶 ↦ 𝐶, (𝑂) ↦ 𝐵𝐶, 𝐴 ↦ 𝐷 ≡ 𝐴𝐼 ∩ 𝐵𝐶, 𝐿 ↦ 𝐸 ≡ 𝐾𝐿 ∩ 𝐵𝐶
Ta có:
𝐿𝐵 = 𝑘 ⋅ 𝐸𝐵
𝐾𝐸 ⋅ 𝐾𝐵; 𝐿𝐶 =
𝑘 ⋅ 𝐸𝐷
𝐾𝐸 ⋅ 𝐾𝐷; 𝐿𝐴 =
𝑘 ⋅ 𝐸𝐶
𝐾𝐸 ⋅ 𝐾𝐶
Vậy nên:
𝐿𝐵 = 𝐿𝐴 + 𝐿𝐶 ⇔ 𝑘 ⋅ 𝐸𝐵
𝐾𝐸 ⋅ 𝐾𝐵 =
𝑘 ⋅ 𝐸𝐷
𝐾𝐸 ⋅ 𝐾𝐷+
𝑘 ⋅ 𝐸𝐶
𝐾𝐸 ⋅ 𝐾𝐶 ⇔
𝐸𝐵


𝐾𝐵 =
𝐸𝐷
𝐾𝐷+
𝐸𝐶
𝐾𝐶 ⇔
𝐸𝐵 − 𝐸𝐶
𝐾𝐵 =
𝐸𝐷
𝐾𝐷
Nhưng 𝐸𝐵 − 𝐸𝐶 = 2𝐸𝑀 nên đẳng thức trên tương đương với:


⇔ 𝐸𝐷
2𝐸𝑀 =
𝐾𝐷
𝐾𝐵 =
𝐵𝐷
𝐵𝐴 =
𝐶𝐷
𝐶𝐴 =
𝐵𝐶
𝐴𝐵 + 𝐴𝐶 ⇔
𝐸𝐷
𝐸𝑀 =
2𝑎
𝑏 + 𝑐 (1)
Ta có:
𝐸𝐵
𝐸𝐶 =
𝑆𝐾𝐵𝐸
𝑆<sub>𝐾𝐶𝐸</sub> =


𝑆𝑁𝐵𝐸
𝑆<sub>𝑁𝐶𝐸</sub> =
𝑆𝐾𝐵𝐸− 𝑆𝑁𝐵𝐸
𝑆<sub>𝐾𝐶𝐸</sub>− 𝑆<sub>𝑁𝐶𝐸</sub> =
𝑆𝐾𝐵𝑁
𝑆<sub>𝐾𝐶𝑁</sub> =


𝐾𝐵 ⋅ 𝐵𝑁 ⋅ sin ∠𝑁𝐵𝐾
𝐾𝐶 ⋅ 𝐶𝑁 ⋅ sin ∠𝑁𝐶𝐾 =


𝐼𝐶
𝐼𝐵⋅


sin ∠𝑁𝐵𝐾
sin ∠𝑁𝐶𝐾
Hơn nữa:


∠𝑁𝐵𝐾 = ∠𝐾𝐵𝐶 − ∠𝑁𝐵𝐶 =1


2(∠𝐵𝐴𝐶 − ∠𝐴𝐶𝐵)
∠𝑁𝐶𝐾 = ∠𝐾𝐶𝐵 − ∠𝑁𝐶𝐵 =1


2(∠𝐵𝐴𝐶 − ∠𝐴𝐵𝐶)


</div>
<span class='text_page_counter'>(35)</span><div class='page_container' data-page=35>

𝐼𝐶
𝐼𝐵=


sin ∠𝐼𝐵𝐶
sin ∠𝐼𝐶𝐵 =



sin𝐵<sub>2</sub>
sin𝐶<sub>2</sub>


=cos
𝐴 + 𝐶


2
cos𝐴 + 𝐵<sub>2</sub>
Nên:
𝐸𝐵
𝐸𝐶 =
𝐼𝐶
𝐼𝐵⋅
sin ∠𝑁𝐵𝐾
sin ∠𝑁𝐶𝐾 =


cos𝐴 + 𝐶<sub>2</sub> sin𝐴 − 𝐶<sub>2</sub>
cos𝐴 + 𝐵<sub>2</sub> sin𝐴 − 𝐵<sub>2</sub>


= sin 𝐴 − sin 𝐶
sin 𝐴 − sin 𝐵 =


𝑎 − 𝑐
𝑎 − 𝑏
Suy ra:
𝐸𝐵
𝑎 − 𝑐=
𝐸𝐶
𝑎 − 𝑏=
𝐸𝐵 + 𝐸𝐶


𝑎 − 𝑐 + 𝑏 − 𝑐 =
𝑎


2𝑎 − 𝑏 − 𝑐 ⇒ 𝐸𝐵 =


𝑎(𝑎 − 𝑐)
2𝑎 − 𝑏 − 𝑐
Suy ra:


𝐸𝑀 = 𝐸𝐵 − 𝑀𝐵 = 𝑎(𝑎 − 𝑐)
2𝑎 − 𝑏 − 𝑐−


𝑎
2=


𝑎(𝑏 − 𝑐)
2(2𝑎 − 𝑏 − 𝑐)
𝐸𝐷 = 𝐸𝐵 − 𝐵𝐷 = 𝑎(𝑎 − 𝑐)


2𝑎 − 𝑏 − 𝑐−
𝑎𝑐
𝑏 + 𝑐 =


𝑎2(𝑏 − 𝑐)
(𝑏 + 𝑐)(2𝑎 − 𝑏 − 𝑐)
Do đó:


𝐸𝐷
𝐸𝑀=



𝑎2(𝑏 − 𝑐)
(𝑏 + 𝑐)(2𝑎 − 𝑏 − 𝑐)⋅


2(2𝑎 − 𝑏 − 𝑐)
𝑎(𝑏 − 𝑐) =


2𝑎
𝑏 + 𝑐
Vậy đẳng thức (1) là đúng và ta có đpcm.


<b>34. Tam giác 𝑨𝑩𝑪 vng tại 𝑨 và bị phủ kín bởi hai đường tròn đơn vị. Chứng minh rằng: </b>
𝑺<sub>𝑨𝑩𝑪</sub>≤𝟑√𝟑


𝟐


<i>Lời giải:</i>


<b>35. Tam giác 𝑨𝑩𝑪 nội tiếp trong đường tròn (𝑶) và có trực tâm 𝑯. 𝑨</b><sub>𝟎</sub>, 𝑩<sub>𝟎</sub>, 𝑪<sub>𝟎</sub><b> là trung điểm </b>
<b>của </b>𝑩𝑪, 𝑪𝑨, 𝑨𝑩. 𝑨𝑨<sub>𝟎</sub>, 𝑩𝑩<sub>𝟎</sub>, 𝑪𝑪<sub>𝟎</sub><b> cắt lại </b>(𝑶) tại 𝑨<sub>𝟏</sub>, 𝑩<sub>𝟏</sub>, 𝑪<sub>𝟏</sub><b>. </b>𝑯𝑨<sub>𝟎</sub>, 𝑯𝑩<sub>𝟎</sub>, 𝑯𝑪<sub>𝟎</sub><b> cắt lại </b>(𝑶) tại
𝑨<sub>𝟐</sub>, 𝑩<sub>𝟐</sub>, 𝑪<sub>𝟐</sub><b>. 𝑺</b><sub>𝒂</sub>≡ 𝑨<sub>𝟏</sub>𝑨<sub>𝟐</sub>∩ 𝑩𝑪, 𝑺<sub>𝒃</sub>≡ 𝑩<sub>𝟏</sub>𝑩<sub>𝟐</sub>∩ 𝑪𝑨, 𝑺<sub>𝒄</sub> ≡ 𝑪<sub>𝟏</sub>𝑪<sub>𝟐</sub>∩ 𝑨𝑩. Chứng minh rằng 𝑺<sub>𝒂</sub>, 𝑺<sub>𝒃</sub>, 𝑺<sub>𝒄</sub>
<b>thẳng hàng. </b>


</div>
<span class='text_page_counter'>(36)</span><div class='page_container' data-page=36>

Ta có:


(𝐴<sub>1</sub>𝑆<sub>𝑎</sub>, 𝐴<sub>1</sub>𝐵) ≡ (𝐴<sub>1</sub>𝑆<sub>𝑎</sub>, 𝐴<sub>1</sub>𝐴) + (𝐴<sub>1</sub>𝐴, 𝐴<sub>1</sub>𝐵) ≡𝜋


2+ (𝐶𝐴, 𝐶𝐵) (mod 𝜋)
(𝐴<sub>1</sub>𝑆<sub>𝑎</sub>, 𝐴<sub>1</sub>𝐶) ≡ (𝐴<sub>1</sub>𝑆<sub>𝑎</sub>, 𝐴<sub>1</sub>𝐴) + (𝐴<sub>1</sub>𝐴, 𝐴<sub>1</sub>𝐶) ≡𝜋


2+ (𝐵𝐴, 𝐵𝐶) (mod 𝜋)


Nên:


sin(𝐴<sub>1</sub>𝑆<sub>𝑎</sub>, 𝐴<sub>1</sub>𝐵)
sin(𝐴<sub>1</sub>𝑆<sub>𝑎</sub>, 𝐴<sub>1</sub>𝐶)=


sin (𝜋<sub>2 +</sub>(𝐶𝐴, 𝐶𝐵))
sin (𝜋<sub>2 +</sub>(𝐵𝐴, 𝐵𝐶))


= cos(𝐶𝐴, 𝐶𝐵)
cos(𝐵𝐴, 𝐵𝐶)
Hơn nữa:


𝐴<sub>1</sub>𝐵
𝐴1𝐶


= sin ∠𝐴0𝐴𝐵
sin ∠𝐴0𝐴𝐶


=𝐴𝐵 ⋅ sin ∠𝐴0𝐴𝐵
𝐴𝐶 ⋅ sin ∠𝐴0𝐴𝐶


⋅𝐴𝐶
𝐴𝐵 =


𝑆<sub>𝐴</sub><sub>0</sub><sub>𝐴𝐵</sub>
𝑆𝐴0𝐴𝐶


⋅𝐴𝐶
𝐴𝐵 =
𝐴𝐶


𝐴𝐵
Suy ra:
𝑆<sub>𝑎</sub>𝐵
̅̅̅̅̅
𝑆𝑎𝐶
̅̅̅̅̅ =


𝑆<sub>𝐴</sub><sub>1</sub><sub>𝑆</sub><sub>𝑎</sub><sub>𝐵</sub>
̅̅̅̅̅̅̅̅
𝑆𝐴1𝑆𝑎𝐶
̅̅̅̅̅̅̅̅ =


𝐴<sub>1</sub>𝑆<sub>𝑎</sub>⋅ 𝐴<sub>1</sub>𝐵 ⋅ sin(𝐴<sub>1</sub>𝑆<sub>𝑎</sub>, 𝐴<sub>1</sub>𝐵)
𝐴1𝑆𝑎⋅ 𝐴1𝐶 ⋅ sin(𝐴1𝑆𝑎, 𝐴1𝐶)


= 𝐴1𝐵
𝐴1𝐶


⋅sin(𝐴1𝑆𝑎, 𝐴1𝐵)
sin(𝐴1𝑆𝑎, 𝐴1𝐶)


= 𝐴𝐶
𝐴𝐵⋅


cos(𝐶𝐴, 𝐶𝐵)
cos(𝐵𝐴, 𝐵𝐶)
Lập các tỉ số tương tự và nhân chúng lại với nhau ta có:


𝑆𝑎𝐵
̅̅̅̅̅


𝑆<sub>𝑎</sub>𝐶
̅̅̅̅̅⋅
𝑆𝑏𝐶
̅̅̅̅̅
𝑆<sub>𝑏</sub>𝐴
̅̅̅̅̅⋅
𝑆𝑐𝐴
̅̅̅̅̅
𝑆<sub>𝑐</sub>𝐵
̅̅̅̅̅=
𝐴𝐶
𝐴𝐵⋅
𝐴𝐵
𝐵𝐶⋅
𝐵𝐶
𝐴𝐶⋅
cos(𝐶𝐴, 𝐶𝐵)
cos(𝐵𝐴, 𝐵𝐶)⋅


cos(𝐵𝐴, 𝐵𝐶)
cos(𝐴𝐵, 𝐴𝐶)⋅


cos(𝐴𝐵, 𝐴𝐶)
cos(𝐶𝐴, 𝐶𝐵)= 1
Theo định lý Menelaus ta có 𝑆𝑎, 𝑆𝑏, 𝑆𝑐 thẳng hàng (đpcm).


<b>36. Tam giác 𝑨𝑩𝑪 nội tiếp trong đường trịn (𝑶) và có trực tâm 𝑯. 𝑨</b><sub>𝟎</sub>, 𝑩<sub>𝟎</sub>, 𝑪<sub>𝟎</sub><b> là trung điểm </b>
<b>của </b>𝑩𝑪, 𝑪𝑨, 𝑨𝑩. 𝑨𝑨<sub>𝟎</sub>, 𝑩𝑩<sub>𝟎</sub>, 𝑪𝑪<sub>𝟎</sub><b> cắt lại </b>(𝑶) tại 𝑨<sub>𝟏</sub>, 𝑩<sub>𝟏</sub>, 𝑪<sub>𝟏</sub><b>. </b>𝑨<sub>𝟎</sub>𝑯, 𝑩<sub>𝟎</sub>𝑯, 𝑪<sub>𝟎</sub>𝑯 cắt lại (𝑶) tại
𝑨𝟐, 𝑩𝟐, 𝑪𝟐<b>. Chứng minh rằng 𝑨</b>𝟏𝑨𝟐, 𝑩𝟏𝑩𝟐, 𝑪𝟏𝑪𝟐<b> đồng qui. </b>



<i>Chứng minh:</i>


S<sub>a</sub>


G


C<sub>1</sub> B1


</div>
<span class='text_page_counter'>(37)</span><div class='page_container' data-page=37>

Đặt 𝐵<sub>3</sub> ≡ 𝐻𝐵<sub>0</sub>∩ (𝑂), 𝐶<sub>3</sub> ≡ 𝐻𝐶<sub>0</sub>∩ (𝑂), ta có 𝐵𝐵<sub>3</sub>, 𝐶𝐶<sub>3</sub> là các đường kính của (𝑂).


Xét lục giác 𝐵𝐶𝐵<sub>1</sub>𝐵<sub>3</sub>𝐶<sub>1</sub>𝐶<sub>3</sub> nội tiếp trong đường tròn (𝑂). Ta có 𝐵𝐵3∩ 𝐶𝐶3 ≡ 𝑂, 𝐵𝐵1∩ 𝐶𝐶1 ≡ 𝐺


và 𝐵1𝐶3∩ 𝐶1𝐵3 ≡ 𝐽 nên theo định lý Pascal thì 𝑂, 𝐺, 𝐽 thằng hàng hay 𝐽 thuộc đường thẳng Euler


của tam giác 𝐴𝐵𝐶.


Lại xét lục giác 𝐵<sub>2</sub>𝐶<sub>2</sub>𝐵<sub>1</sub>𝐵<sub>3</sub>𝐶<sub>3</sub>𝐶<sub>1</sub> nội tiếp trong (𝑂) , ta có 𝐵<sub>2</sub>𝐵<sub>3</sub>∩ 𝐶<sub>2</sub>𝐶<sub>3</sub> ≡ 𝐻, 𝐵<sub>2</sub>𝐵<sub>1</sub>∩ 𝐶<sub>2</sub>𝐶<sub>1</sub> ≡ 𝐼<sub>𝑎</sub>
và 𝐶<sub>3</sub>𝐵<sub>1</sub>∩ 𝐵<sub>3</sub>𝐶<sub>1</sub> ≡ 𝐽 nên cũng theo định lý Pascal thì 𝐻, 𝐽, 𝐼<sub>𝑎</sub> thẳng hàng.


Suy ra giao điểm của 𝐵1𝐵2 và 𝐶1𝐶2 thuộc đường thẳng Euler của tam giác 𝐴𝐵𝐶.


Lý luận tương tự ta cũng có 𝐴<sub>1</sub>𝐴<sub>2</sub>, 𝐵<sub>1</sub>𝐵<sub>2</sub>, 𝐶<sub>1</sub>𝐶<sub>2</sub> đồng quy tại một điểm nằm trên đường thẳng
Euler của tam giác 𝐴𝐵𝐶 (đpcm).


<b>37. Cho tam giác </b>𝑨𝑩𝑪 cân tại 𝑨. Đường tròn (𝑶) tiếp xúc với 𝑨𝑩, 𝑨𝑪 tại 𝑩, 𝑪. 𝑸 là một
<b>điểm tùy ý nằm trong góc ∠𝑩𝑨𝑪. 𝑶𝑷 ⊥ 𝑨𝑸 tại 𝑷. Đặt 𝑲 ≡ 𝑶𝑷 ∩ (𝑩𝑷𝑸), 𝑳 ≡ 𝑶𝑷 ∩ (𝑪𝑷𝑸). </b>
<b>Chứng minh rằng 𝑶𝑲 = 𝑶𝑳. </b>


<i>Lời giải: </i>



J


C<sub>3</sub> B<sub>3</sub>


G


I


A<sub>1</sub>
C<sub>2</sub>


B<sub>1</sub>


B<sub>2</sub>
C<sub>1</sub>


A<sub>2</sub>


A<sub>0</sub>
B<sub>0</sub>
C<sub>0</sub>


H


O
A


</div>
<span class='text_page_counter'>(38)</span><div class='page_container' data-page=38>

Xét phép nghịch đảo sau:


ℐ1(𝐴, 𝑘1 = 𝐴𝑃̅̅̅̅ ⋅ 𝐴𝑄̅̅̅̅): 𝑃 ↦ 𝑄, 𝐵 ↦ 𝐵′≡ 𝐴𝐵 ∩ (𝐵𝑃𝑄), 𝐶 ↦ 𝐶′≡ 𝐴𝐶 ∩ (𝐶𝑃𝑄)



Ta có 𝐴𝐵 = 𝐴𝐶 nên 𝐴𝐵′<sub>= 𝐴𝐶</sub>′<sub>, suy ra </sub><sub>𝐵</sub>′<sub>𝐶</sub>′<sub>∥ 𝐵𝐶. Hơn nữa 𝐴, 𝐵, 𝐶, 𝑃 đồng viên nên </sub><sub>𝐵</sub>′<sub>, 𝑄, 𝐶′ </sub>


thẳng hàng. Ký hiệu 𝑀 ≡ 𝑄𝑂 ∩ (𝐵𝑃𝑄), 𝑁 ≡ 𝑄𝑂 ∩ (𝐶𝑃𝑄), 𝐼 ≡ 𝑄𝐵 ∩ (𝑂), 𝐽 ≡ 𝑄𝐶 ∩ (𝑂) và
𝑅 ≡ 𝐵𝐶 ∩ (𝐶𝑃𝑄). Ta có 𝐶𝑅𝑄𝐶′ là hình thang và nội tiếp trong đường trịn (𝐶𝑃𝑄) nên 𝐶𝑅𝑄𝐶′ là
hình thang cân, dẫn đến 𝑅𝐶′<sub>= 𝐶𝑄. Từ đó: </sub>


(𝑃𝐶, 𝑃𝑄) ≡ (𝐵𝐶, 𝐵𝐴) ≡ (𝐶𝐴, 𝐶𝐵) ≡ (𝐼𝐶, 𝐼𝑄) (mod 𝜋)
Suy ra 𝑃, 𝑁, 𝐿, 𝐶′, 𝑄 đồng viên và tương tự thì 𝑃, 𝑀, 𝐾, 𝐵′, 𝑄 cũng đồng viên.
Lại xét phép nghịch đảo ℐ2(𝑄, 𝑘2 = 𝒫𝑄/(𝑂)). Ta có ℐ2: 𝐵 ↦ 𝐼, 𝐶 ↦ 𝐽 nên:


ℐ2: (𝐵𝑃𝑄) ↦ 𝐶𝐼, (𝐶𝑃𝑄) ↦ 𝐵𝐽, 𝑂 ↦ 𝑂′, 𝑃 ↦ 𝑃 ≡ 𝐵𝐽 ∩ 𝐶𝐼, 𝑀 ↦ 𝑀′, 𝑁 ↦ 𝑁′


Theo tính chất của phép nghịch đảo thì vì 𝑂𝑃 ⊥ 𝐴𝑄 nên 𝑃′<sub>𝑂</sub>′<sub>⊥ 𝑀</sub>′<sub>𝑁</sub>′<sub>. </sub>


Từ đó:
𝑂′<sub>𝑀</sub>′


𝑂′<sub>𝑁</sub>′ =


𝑃′<sub>𝑀</sub>′<sub>⋅ cos ∠𝑃</sub>′<sub>𝑀</sub>′<sub>𝑁</sub>′


𝑃′<sub>𝑁</sub>′<sub>⋅ cos ∠𝑃</sub>′<sub>𝑁</sub>′<sub>𝑀</sub>′ =


sin ∠𝑃′<sub>𝑁</sub>′<sub>𝑀</sub>′


sin ∠𝑃′<sub>𝑀</sub>′<sub>𝑁</sub>′⋅


cos ∠𝑀𝑃𝑄
cos ∠𝑁𝑃𝑥 =



sin ∠𝑁𝑃𝑄
sin ∠𝑀𝑃𝑄⋅


sin ∠𝑀𝑃𝑂
sin ∠𝑁𝑃𝑂
= 𝑁𝑃 ⋅ sin ∠𝑁𝑃𝑄


𝑀𝑃 ⋅ sin ∠𝑀𝑃𝑄⋅


𝑀𝑃 ⋅ sin ∠𝑀𝑃𝑂
𝑁𝑃 ⋅ sin ∠𝑁𝑃𝑂 =


𝑆<sub>Δ𝑃𝑁𝑄</sub>
𝑆<sub>Δ𝑃𝑀𝑄</sub>⋅


𝑆Δ𝑃𝑀𝑂


𝑆<sub>Δ𝑃𝑁𝑂</sub> =


𝑆<sub>Δ𝑃𝑁𝑄</sub>
𝑆<sub>Δ𝑃𝑀𝑄</sub>=


𝑄𝑁
𝑄𝑀=


𝑄𝑀′
𝑄𝑁′


Vậy:



𝑂𝑁 = 𝑘2⋅ 𝑂


′<sub>𝑁</sub>′


𝑄𝑂′<sub>⋅ 𝑄𝑁</sub>′=


𝑘<sub>2</sub> ⋅ 𝑂′𝑀′


𝑄𝑂′<sub>⋅ 𝑄𝑀</sub>′= 𝑂𝑀


x


C'
B'
R


N'
M'


O'
P


K


L


P'
I



J
M


N


A
O


B


C


</div>
<span class='text_page_counter'>(39)</span><div class='page_container' data-page=39>

Mặt khác (𝐾𝑀, 𝐾𝑃) ≡ (𝐶𝑀, 𝐶𝑃) ≡ (𝐿𝑁, 𝐿𝑃) (mod 𝜋) nên 𝐾𝑀 ∥ 𝐿𝑁. Vậy nên 𝑀𝐾𝑁𝐿 là hình
bình hành. Do đó, trung điểm 𝑂 của 𝑀𝑁 cũng là trung điểm của 𝐾𝐿, tức là 𝑂𝐾 = 𝑂𝐿 (đpcm).
<b>38. Tam giác 𝑨𝑩𝑪 có ∠𝑨𝑪𝑩 = 𝟐∠𝑨𝑩𝑪. Điểm 𝑷 nằm trong góc ∠𝑩𝑨𝑪 sao cho 𝑷𝑩 = 𝑷𝑪 và </b>
𝑨𝑪 = 𝑨𝑷. Chứng minh rằng ∠𝑷𝑨𝑪 = 𝟐∠𝑷𝑨𝑩.


<i>Lời giải:</i>


Gọi 𝐷 là chân đường phân giác góc ∠𝐴𝐶𝐵 của tam giác 𝐴𝐵𝐶. Ta có:
∠𝐷𝐶𝐵 =1


2∠𝐴𝐶𝐵 = ∠𝐴𝐵𝐶


Nên 𝐷𝐶 = 𝐷𝐵, hơn nữa 𝑃𝐶 = 𝑃𝐵 nên 𝑃𝐷 là trung trực của 𝐵𝐶. Gọi 𝐸 là điểm đối xứng với 𝐶
qua 𝐴𝑃. Ta lại có 𝑃𝐶 = 𝑃𝐸 = 𝑃𝐵 nên 𝑃 là tâm đường tròn (𝐸𝐵𝐶).


Đường thẳng qua 𝑃 và vng góc với 𝐸𝐵 cắt 𝐵𝐶 tại 𝑄, ta có:
∠𝐸𝐶𝐵 =1



2∠𝐸𝑃𝐵 = ∠𝑄𝑃𝐵 = ∠𝑄𝑃𝐸


Hơn nữa 𝐴𝑃 ⊥ 𝐶𝐸 và 𝑃𝐷 ⊥ 𝐵𝐶 nên ∠𝐸𝐶𝐵 = ∠𝐴𝑃𝐷. Suy ra ∠𝐴𝑃𝐸 = ∠𝑄𝑃𝐸. Từ đó:
∠𝐴𝐶𝑃 = ∠𝐴𝑃𝐸 = ∠𝐴𝑃𝐷 + ∠𝐷𝑃𝐸 = ∠𝐷𝑃𝐸 + ∠𝑄𝑃𝐸 = ∠𝐷𝑃𝑄
Mặt khác 𝑃𝑄 ⊥ 𝐵𝐸 và 𝑃𝐷 ⊥ 𝐵𝐶 nên ∠𝐴𝐶𝑃 = ∠𝐷𝑃𝑄 = ∠𝐸𝐵𝑄. Tức là:


∠𝐴𝐶𝐷 + ∠𝐷𝐶𝑃 = ∠𝐷𝐵𝑃 + ∠𝐸𝐵𝐴 − ∠𝑃𝐵𝐶
Nhưng ∠𝐷𝐶𝑃 = ∠𝐷𝐵𝑃 nên:


∠𝐸𝐵𝐴 = ∠𝐴𝐶𝐷 + ∠𝑃𝐵𝐶 = 1


2∠𝐴𝐶𝐵 + ∠𝑃𝐵𝐶 = ∠𝐴𝐵𝐶 + ∠𝑃𝐵𝐶 = ∠𝐴𝐵𝑃
Suy ra:


𝐴𝐵
sin ∠𝐴𝐸𝐵=


𝐴𝐸
sin ∠𝐴𝐵𝐸 =


𝐴𝐹
sin ∠𝐴𝐵𝑃 =


𝐴𝐵
sin ∠𝐴𝑃𝐵


Dẫn đến sin ∠𝐴𝐸𝐵 = sin ∠𝐴𝑃𝐵, suy ra ∠𝐴𝐸𝐵 = ∠𝐴𝑃𝐵 hoặc ∠𝐴𝐸𝐵 + ∠𝐴𝑃𝐵 = 180°.


Q
E



P
A


C B


</div>
<span class='text_page_counter'>(40)</span><div class='page_container' data-page=40>

Nhưng nếu ∠𝐴𝐸𝐵 + ∠𝐴𝑃𝐵 = 180° thì 𝐴, 𝐸, 𝑃, 𝐵 đồng viên, suy ra:


(𝐶𝑃, 𝐶𝐴) ≡ (𝐸𝐴, 𝐸𝑃) ≡ (𝐵𝐴, 𝐵𝑃) ≡ (𝐶𝑃, 𝐶𝐷) (mod 𝜋)
Từ đó ta có 𝐶, 𝐴, 𝐷 thẳng hàng (vô lý).


Vậy ∠𝐴𝐸𝐵 = ∠𝐴𝑃𝐵, suy ra tam giác 𝐴𝐸𝐵 và tam giác 𝐴𝑃𝐵 bằng nhau (g.c.g). Từ đó:
∠𝑃𝐴𝐵 = ∠𝐵𝐴𝐸 =1


2∠𝑃𝐴𝐸 =
1


2∠𝑃𝐴𝐶
Hay ∠𝑃𝐴𝐶 = 2∠𝑃𝐴𝐵 (đpcm).


<b>39. Cho tam giác 𝑨𝑩𝑪 và 𝑲 ∈ 𝑩𝑪 sao cho 𝑲𝑩 = 𝟐𝑲𝑪. Gọi 𝑳 là hình chiếu của 𝑩 trên 𝑨𝑲 và </b>
𝑭 là trung điểm của 𝑩𝑪. Giả sử rằng ∠𝑲𝑨𝑩 = 𝟐∠𝑲𝑨𝑪. Chứng minh rằng 𝑭𝑳 ⊥ 𝑨𝑪.


<i>Lời giải:</i>


Lấy 𝐷 là điểm đối xứng với 𝐶 qua 𝐴𝐾, 𝐼 ≡ 𝐴𝐾 ∩ 𝐶𝐷 và 𝐻 ≡ 𝐷𝐿 ∩ 𝐴𝐶. Ta có:
2 =𝐾𝐵


𝐾𝐶 =
𝑆<sub>Δ𝐴𝐾𝐵</sub>


𝑆<sub>Δ𝐴𝐾𝐶</sub> =


𝐴𝐵 ⋅ sin 2∠𝐾𝐴𝐶
𝐴𝐶 ⋅ sin ∠𝐾𝐴𝐶 =


2 ⋅ 𝐴𝐵 ⋅ sin ∠𝐾𝐴𝐶 ⋅ cos 𝐾𝐴𝐶
𝐴𝐶 ⋅ sin ∠𝐾𝐴𝐶


Suy ra:


𝐴𝐷


𝐴𝐵 = cos ∠𝐾𝐴𝐶 = cos ∠𝐷𝐴𝐵
Vậy ∠𝐴𝐷𝐵 = 90° = ∠𝐴𝐿𝐵 nên 𝐴, 𝐵, 𝐷, 𝐿 đồng viên.


Mặt khác 𝐴𝐾 ⊥ 𝐶𝐷 và 𝐵𝐿 ⊥ 𝐴𝐾 nên 𝐵𝐿 ∥ 𝐶𝐷, dẫn đến ∠𝐻𝐷𝐼 = ∠𝐵𝐿𝐷 = ∠𝐵𝐴𝐷 = ∠𝐻𝐴𝐼. Từ
đây lại suy ra 𝐴, 𝐻, 𝐼, 𝐷 đồng viên và do đó ∠𝐴𝐻𝐷 = ∠𝐴𝐼𝐷 = 90°, tức là 𝐷𝐿 ⊥ 𝐴𝐶.


I


D


H


F


L


K
B



A


</div>
<span class='text_page_counter'>(41)</span><div class='page_container' data-page=41>

Mà 𝐴𝐿 ⊥ 𝐶𝐷 nên 𝐿 là trực tâm của tam giác 𝐴𝐶𝐷, suy ra 𝐶𝐿 ⊥ 𝐴𝐷, hơn nữa 𝐵𝐷 ⊥ 𝐴𝐷 nên 𝐵𝐷 ∥
𝐶𝐿, mặt khác 𝐵𝐿 ∥ 𝐶𝐷 nên 𝐵𝐿𝐶𝐷 là hình bình hành. Do vậy, trung điểm 𝐹 của 𝐵𝐶 cũng là trung
điểm của 𝐷𝐿 và từ đây ta có 𝐹𝐿 ⊥ 𝐴𝐶 (đpcm).


<b>40. Đường tròn (I) nội tiếp tam giác </b>𝑨𝑩𝑪, tiếp xúc với các cạnh 𝑩𝑪, 𝑪𝑨, 𝑨𝑩 tại 𝑿, 𝒀, 𝒁.
𝑿′, 𝒀′, 𝒁′<b> lần lượt là các điểm trên các tia </b>𝑰𝑿, 𝑰𝒀, 𝑰𝒁 sao cho 𝑰𝑿′ = 𝑰𝒀′ = 𝑰𝒁′<b>. Chứng minh </b>
<b>rằng 𝑨𝑿</b>′, 𝑩𝒀′, 𝑪𝒁′<b> đồng quy. </b>


<i>Lời giải:</i>


Xét phép đối xứng trục 𝐴𝐼 biến 𝑍 ↦ 𝑌, 𝐼𝑍 = 𝐼𝑌, 𝐼𝑍′<sub>= 𝐼𝑌</sub>′<sub> nên 𝑍</sub>′<sub>↦ 𝑌</sub>′<sub>, suy ra ∠𝑍𝐴𝑍</sub>′<sub>= ∠𝑌𝐴𝑌</sub>′


Tương tự ta cũng có ∠𝑍𝐵𝑍′ = ∠𝑋𝐵𝑋′ và ∠𝑋𝐶𝑋′ = ∠𝑌𝐶𝑌′. Từ đó suy ra ∠𝐴𝐵𝑋′= ∠𝐶𝐵𝑍′,
∠𝐶𝐴𝑍′<sub>= ∠𝐵𝐴𝑌</sub>′<sub> và ∠𝐵𝐶𝑌</sub>′<sub>= ∠𝐴𝐶𝑋</sub>′<sub>. </sub>


Suy ra:


I'


X'
X


Y'


Y


Z



I
A


B C


</div>
<span class='text_page_counter'>(42)</span><div class='page_container' data-page=42>

sin ∠𝑋′𝐴𝐵
sin ∠𝑋′<sub>𝐴𝐶</sub> =


𝑅<sub>𝑋</sub>′<sub>𝐴𝐶</sub>
sin ∠𝑋′<sub>𝐴𝐶</sub>


𝑅<sub>𝑋</sub>′<sub>𝐴𝐵</sub>
sin ∠X′<sub>AB</sub>


⋅𝑅𝑋′𝐴𝐵
𝑅<sub>𝑋</sub>′<sub>𝐴𝐶</sub> =


𝐴𝑋′


sin ∠𝐴𝐶𝑋′


𝐴𝑋′


sin ∠𝐴𝐵𝑋′


⋅𝑋


′<sub>𝐶</sub>



𝑋′<sub>𝐵</sub>=


sin ∠𝐴𝐵𝑋′
sin ∠𝐴𝐶𝑋′⋅


sin ∠𝑋′𝐵𝐶
sin ∠𝑋′<sub>𝐶𝐵</sub>


Lý luận tương tự ta có:
sin ∠𝑌′𝐵𝐶
sin ∠𝑌′<sub>𝐵𝐴</sub>=


sin ∠𝐵𝐶𝑌′
sin ∠𝐵𝐴𝑌′⋅


sin ∠𝑌′𝐶𝐴
sin ∠𝑌′<sub>𝐴𝐵</sub> và


sin ∠𝑍′𝐶𝐴
sin ∠𝑍′<sub>𝐶𝐵</sub>=


sin ∠𝐶𝐴𝑍′
sin ∠𝐶𝐵𝑍′⋅


sin ∠𝑍′𝐴𝐵
sin ∠𝑍′<sub>𝐵𝐴</sub>


Nhân các tỉ số trên lại với nhau suy ra:
sin ∠𝑋′<sub>𝐴𝐵</sub>



sin ∠𝑋′<sub>𝐴𝐶</sub>⋅


sin ∠𝑌′<sub>𝐵𝐶</sub>


sin ∠𝑌′<sub>𝐵𝐴</sub>⋅


sin ∠𝑍′<sub>𝐶𝐴</sub>


sin ∠𝑍′<sub>𝐶𝐵</sub>


= (sin ∠𝐴𝐵𝑋




sin ∠𝐴𝐶𝑋′⋅


sin ∠𝐵𝐶𝑌′


sin ∠𝐵𝐴𝑌′⋅


sin ∠𝐶𝐴𝑍′


sin ∠𝐶𝐵𝑍′) ⋅ (


sin ∠𝑋′<sub>𝐵𝐶</sub>


sin ∠𝑋′<sub>𝐶𝐵</sub>⋅


sin ∠𝑌′<sub>𝐶𝐴</sub>



sin ∠𝑌′<sub>𝐴𝐵</sub>


sin ∠𝑍′<sub>𝐴𝐵</sub>


sin ∠𝑍′<sub>𝐵𝐴</sub>)


= 1


Vậy theo định lý Ceva thì 𝐴𝑋′, 𝐵𝑌′, 𝐶𝑍′ cũng đồng qui (đpcm).
<b>41. Lục giác 𝑨𝑩𝑪𝑫𝑬𝑭 có độ dài các cạnh bằng nhau đồng thời: </b>


∠𝑨 + ∠𝑪 + ∠𝑬 = ∠𝑩 + ∠𝑫 + ∠𝑭
<b>Chứng minh rằng ∠𝑨 = ∠𝑫, ∠𝑩 = ∠𝑬, ∠𝑪 = ∠𝑭. </b>


<i>Lời giải:</i>


Xét phép quay 𝑅 (𝐶, 𝛼 = (𝐶𝐵⃗⃗⃗⃗⃗ , 𝐶𝐷⃗⃗⃗⃗⃗ )) : 𝐵 ↦ 𝐷, 𝐴 ↦ 𝐾. Ta có:


360° = ∠𝐴𝐹𝐸 + ∠𝐴𝐵𝐶 + ∠𝐶𝐷𝐸 = ∠𝐸𝐷𝐾 + ∠𝐶𝐷𝐾 + ∠𝐶𝐷𝐸


Mà ∠𝐶𝐷𝐾 = ∠𝐴𝐵𝐶 nên ∠𝐴𝐹𝐸 = ∠𝐸𝐹𝐴. Mặt khác 𝐸𝐾 = 𝐸𝐴, 𝐶𝐴 = 𝐶𝐾 suy ra 𝐷𝐸 = 𝐷𝐶 = 𝐷𝐾
Từ đó ∠𝐵𝐶𝐷 = ∠𝐴𝐶𝐾 = 2∠𝐸𝐶𝐾 = ∠𝐸𝐷𝐾 = ∠𝐴𝐹𝐸, tức là ∠𝐶 = ∠𝐹.


Chứng minh tương tự ta có ∠𝐴 = ∠𝐷 và ∠𝐵 = ∠𝐸 (đpcm).


</div>
<span class='text_page_counter'>(43)</span><div class='page_container' data-page=43>

<b>Hãy tìm vị trí của 𝑴, 𝑵 sao cho 𝑴𝑵 có độ dài nhỏ nhất. </b>


<i>Lời giải:</i>


Lấy điểm 𝐾 sao cho tứ giác lồi 𝐴𝐷𝐾𝐶 là hình bình hành và lấy 𝐻 ∈ 𝐵𝐾 sao cho:


𝐵𝐻


̅̅̅̅
𝐵𝐾
̅̅̅̅ =


𝐵𝑁
̅̅̅̅
𝐵𝐶
̅̅̅̅ =


𝐷𝑀
̅̅̅̅̅
𝐷𝐴
̅̅̅̅
Theo định lý Thales thì 𝐻𝑁 ∥ 𝐶𝐾 ∥ 𝐴𝐷, hơn nữa:


𝑁𝐻
̅̅̅̅̅
𝐴𝐷
̅̅̅̅ =


𝑁𝐻
̅̅̅̅̅
𝐶𝐾
̅̅̅̅ =


𝐵𝑁
̅̅̅̅
𝐵𝐶


̅̅̅̅ =


𝐷𝑀
̅̅̅̅̅
𝐴𝐷
̅̅̅̅


Suy ra 𝑁𝐻 = 𝐷𝑀 tức 𝑀𝐷𝐻𝑁 là hình bình hành, dẫn đến 𝐷𝐻 = 𝑀𝑁.


Vậy 𝑀𝑁 nhỏ nhất khi và chỉ khi 𝐷𝐻 nhỏ nhất, tức là 𝐷𝐻 ⊥ 𝐵𝐾. Từ đó ta xác định được 𝑀, 𝑁.
<b>43. Tứ giác 𝑨𝑩𝑪𝑫 ngoại tiếp đường tròn (𝑰) và </b>𝑨𝑪 ∩ 𝑩𝑫 ≡ 𝑶. 𝑯<sub>𝟏</sub>, 𝑯<sub>𝟐</sub>, 𝑯<sub>𝟑</sub>, 𝑯<sub>𝟒</sub><b> là trực tâm </b>
<b>các tam giác 𝑰𝑨𝑩, 𝑰𝑩𝑪, 𝑰𝑪𝑫, 𝑰𝑫𝑨. Chứng minh rằng 𝑶, 𝑯</b>𝟏, 𝑯𝟐, 𝑯𝟑, 𝑯𝟒<b> thẳng hàng. </b>


<i>Lời giải:</i>


M


H


K


B


C


D
A


</div>
<span class='text_page_counter'>(44)</span><div class='page_container' data-page=44>

Trước hết ta nhắc lại kết quả quen thuộc:
𝑂𝐵


𝑂𝐷=


𝐵𝑀
𝑄𝐷


O
H<sub>4</sub>


H<sub>3</sub>


H<sub>2</sub>


H<sub>1</sub>
D


C
B
A


I
Q


M


N


</div>
<span class='text_page_counter'>(45)</span><div class='page_container' data-page=45>

Ta có: ∠𝐻1𝐵𝑀 = ∠𝑀𝐼𝐴 = ∠𝑄𝐼𝐴 = ∠𝐻4𝐷𝑄 và ∠𝐵𝑀𝐻1 = ∠𝐷𝑄𝐻4 = 90° nên tam giác 𝑀𝐻1𝐵


và tam giác 𝐷𝑄𝐻<sub>4</sub> đồng dạng.
Đặt 𝑂′<sub>≡ 𝐻</sub>



1𝐻4∩ 𝐵𝐷. Từ tam giác đồng dạng trên ta có:


𝑂′<sub>𝐵</sub>


𝑂′<sub>𝐷</sub> =


𝐻<sub>1</sub>𝐵
𝐻<sub>1</sub>𝐷 =


𝐵𝑀
𝑄𝐷 =


𝑂𝐵
𝑂𝐷


Suy ra 𝑂 ≡ 𝑂′. Vậy 𝐻1𝐻4 đi qua 𝑂. Tương tự ta cũng có 𝐻1𝐻2, 𝐻2𝐻3 đều đi qua 𝑂. Từ đó ta có


kết luận của bài toán.


<b>44. Các điểm 𝑫, 𝑬, 𝑭 nằm trên các cạnh 𝑩𝑪, 𝑪𝑨, 𝑨𝑩 của tam giác </b>𝑨𝑩𝑪 sao cho 𝑨𝑫, 𝑩𝑬, 𝑪𝑭
<b>đồng quy tại 𝑶. Gọi </b>𝑯 là hình chiếu của 𝑫 trên 𝑬𝑭 và 𝑿, 𝒀, 𝒁, 𝑻 là hình chiếu của 𝑯 trên
𝑨𝑬, 𝑨𝑭, 𝑶𝑬, 𝑶𝑭. Chứng minh rằng 𝑿, 𝒀, 𝒁, 𝑻 đồng viên.


</div>
<span class='text_page_counter'>(46)</span><div class='page_container' data-page=46>

Đặt 𝐾 ≡ 𝐸𝐹 ∩ 𝐵𝐶 và 𝐽 ≡ 𝐴𝑂 ∩ 𝐸𝐹. Ta có:


(𝐻𝐴, 𝐻𝑂, 𝐻𝐸, 𝐻𝐷) = 𝐻(𝐴𝑂𝐽𝐷) = 𝐾(𝐴𝑂𝐽𝐷) = −1
Mà 𝐻𝐷 ⊥ 𝐻𝐸 nên 𝐻𝐸 là phân giác ∠𝐴𝐻𝑂. Từ đó:


(𝑌𝑋, 𝑌𝑇) ≡ (𝑌𝑋, 𝑌𝐻) + (𝑌𝐻, 𝑌𝑇) ≡ (𝐴𝑋, 𝐴𝐻) + (𝐹𝐻, 𝐹𝑇)



≡ (𝐸𝐴, 𝐸𝐻) + (𝐻𝐸, 𝐻𝐴) + (𝐹𝐻, 𝐹𝑇) ≡ (𝑍𝑋, 𝑍𝐻) + (𝐻𝑂, 𝐻𝐸) + (𝐹𝐻, 𝐹𝑇)
≡ (𝑍𝑋, 𝑍𝐻) + (𝑂𝐻, 𝑂𝑇) ≡ (𝑍𝑋, 𝑍𝐻) + (𝑍𝐻, 𝑍𝑇) ≡ (𝑍𝑋, 𝑍𝑇)(mod 𝜋)
Suy ra 𝑋, 𝑌, 𝑍, 𝑇 đồng viên (đpcm).


<b>45. Cho lục giác </b>𝑨𝑩𝑪𝑫𝑬𝑭 nội tiếp trong (𝑶). Các cạnh của các tam giác 𝚫𝑨𝑪𝑬 và 𝚫𝑩𝑫𝑭
<b>cắt nhau tại sáu điểm </b>𝑴, 𝑵, 𝑷, 𝑸, 𝑹, 𝑺 theo thứ tự đó. Chứng minh rằng 𝑴𝑸, 𝑷𝑺, 𝑵𝑹 đồng
<b>qui. </b>


J
K


O


T


Z


X


Y


H


D


E


F



A


B C


</div>
<span class='text_page_counter'>(47)</span><div class='page_container' data-page=47>

<i>Lời giải: </i>


Đặt 𝑋 ≡ 𝐴𝐷 ∩ 𝐵𝐸, 𝑌 ≡ 𝐶𝐹 ∩ 𝐴𝐷 và 𝑍 ≡ 𝐵𝐸 ∩ 𝐷𝐹. Theo định lý Pascal thì 𝑀, 𝑋, 𝑄, 𝑃, 𝑆, 𝑌 và
𝑅, 𝑍, 𝑁 là các bộ ba điểm thẳng hàng.


Xét tam giác Δ𝑋𝐸𝐷 có 𝐷𝐹, 𝐶𝐸, 𝑋𝑄 đồng qui. theo định lý Ceva ta có:
sin ∠𝑄𝑋𝐸


sin ∠𝑄𝑋𝐷⋅


sin ∠𝐴𝐷𝐹
sin ∠𝐸𝐷𝐹⋅


sin ∠𝐶𝐸𝐷


sin ∠𝐶𝐸𝐵 = 1 ⇒


sin ∠𝑄𝑋𝐸
sin ∠𝑄𝑋𝐷=


sin ∠𝐸𝐷𝐹
sin ∠𝐴𝐷𝐹⋅


sin ∠𝐶𝐸𝐵
sin ∠𝐶𝐸𝐷 =



𝐸𝐹
𝐴𝐹⋅


𝐶𝐵
𝐶𝐷
Lập các tỉ số tương tự và nhân chúng lại với nhau, áp dụng định lý Ceva lần nữa cho tam giác
Δ𝑋𝑌𝑍, ta có 𝑋𝑄, 𝑌𝑆, 𝑍𝑁 đồng qui (đpcm).


<b>46. Đường tròn (𝑶</b>𝟏) và (𝑶𝟐) cắt nhau tại hai điểm 𝑷, 𝑸. Một đường tròn (𝑶) đi qua 𝑷 và


<b>cắt </b>𝑷𝑸, (𝑶𝟏), (𝑶𝟐) tại 𝑨, 𝑩, 𝑪. 𝑿, 𝒀 ∈ (𝑶) sao cho 𝑩𝑿 ∥ 𝑪𝒀 ∥ 𝑨𝑷. 𝑷𝑿, 𝑷𝒀 cắt (𝑶𝟐), (𝑶𝟏) lần


<b>nữa tại 𝒁, 𝑻. Chứng minh rằng 𝑿𝒀 ∥ 𝒁𝑻. </b>


<i>Lời giải:</i>


Y
Z


X
S


R


Q


P
N


M



O
A


B


C


D
E


</div>
<span class='text_page_counter'>(48)</span><div class='page_container' data-page=48>

Xét phép đối xứng trục 𝑂1𝑂2 biến 𝐵 ↦ 𝑈 ∈ (𝑂1), 𝐶 ↦ 𝑉 ∈ (𝑂2). Do phép đối xứng có trục là


trung trực của 𝐴𝑃 biến 𝑋 ↦ 𝐵 và 𝑌 ↦ 𝐶 nên 𝑋𝑈⃗⃗⃗⃗⃗ = 𝐴𝑄⃗⃗⃗⃗⃗ = 𝑌𝑉⃗⃗⃗⃗⃗ <sub>, suy ra 𝑈𝑉 ∥ 𝑋𝑌. </sub>
Ký hiệu 𝑍 ≡ 𝑈𝑉 ∩ (𝑂2), 𝑇 ≡ 𝑈𝑉 ∩ (𝑂1), 𝑀 ≡ 𝑃𝑋 ∩ 𝑄𝑈, 𝑁 ≡ 𝑃𝑌 ∩ 𝑄𝑉. Ta có:


(𝑄𝑀, 𝑄𝑁) ≡ (𝐴𝑋, 𝐴𝑌) ≡ (𝑃𝑀, 𝑃𝑁) (mod 𝜋)
Suy ra 𝑀, 𝑁, 𝑃, 𝑄 đồng viên. Từ đó:


(𝑃𝑀, 𝑃𝑄) ≡ (𝑁𝑀, 𝑁𝑄) ≡ (𝑉𝑍, 𝑉𝑄) ≡ (𝑃𝑍, 𝑃𝑄) (mod 𝜋)


Suy ra 𝑃, 𝑀, 𝑍 thẳng hàng và tương tự thì 𝑃, 𝑁, 𝑇 thẳng hàng. Vậy các điểm 𝑍, 𝑇 ở đây chính là
các điểm 𝑍, 𝑇 được nói đến trong đề bài và ta có đpcm.


<b>47. Lục giác </b>𝑨𝑩𝑪𝑫𝑬𝑭 thỏa mãn các điều kiện: tam giác 𝑨𝑩𝑭 vuông cân tại 𝑨, 𝑩𝑪𝑬𝑭 là
<b>hình bình hành, 𝑨𝑫 = 𝟑, 𝑩𝑪 = 𝟏, 𝑫𝑪 + 𝑫𝑬 = 𝟐√𝟐. Tính diện tích lục giác 𝑨𝑩𝑪𝑫𝑬𝑭. </b>


<i>Lời giải:</i>


A



M
N


X


Y
T


Z
V
U


C
B


Q


P
O<sub>1</sub>


O<sub>2</sub>


</div>
<span class='text_page_counter'>(49)</span><div class='page_container' data-page=49>

Xét phép tịnh tiến 𝒯<sub>𝐵𝐶</sub><sub>⃗⃗⃗⃗⃗ </sub>: 𝐴 ↦ 𝐾, 𝐵 ↦ 𝐶, 𝐹 ↦ 𝐸. Ta có tam giác 𝐶𝐾𝐸 vng cân tại 𝐾 nên:
𝐾𝐶 = 𝐾𝐸 = 1


√2𝐶𝐸
Theo bất đẳng thức Ptoleme trong tứ giác 𝐶𝐾𝐸𝐷 ta có:


𝐾𝐶 ⋅ 𝐷𝐸 + 𝐾𝐸 ⋅ 𝐷𝐶 ≥ 𝐾𝐷 ⋅ 𝐶𝐸 ⇔ 𝐷𝐸 + 𝐷𝐶 ≥ 𝐾𝐷 ⋅𝐶𝐸



𝐾𝐸 ⇔ 𝐾𝐷 ≤ 2
Mặt khác 𝐴𝐾 = 𝐵𝐶 = 1 nên:


𝐴𝐷 ≤ 𝐴𝐾 + 𝐾𝐵 ≤ 3 = 𝐴𝐷


Do đó, 𝐴, 𝐾, 𝐷 thẳng hàng và bất đẳng thức Ptoleme phải xảy ra đẳng thức, tức là 𝐶, 𝐾, 𝐸, 𝐷 đồng
viên, suy ra ∠𝐶𝐷𝐸 = ∠𝐶𝐾𝐸 = 90°.


Đặt góc nhọn giữa hai đường thẳng 𝐾𝐷, 𝐶𝐸 là 𝛼 thì ta có:


𝑆<sub>𝐴𝐵𝐶𝐷𝐸𝐹</sub> = 𝑆<sub>𝐴𝐵𝐹</sub> + 𝑆<sub>𝐵𝐶𝐸𝐹</sub>+ 𝑆<sub>𝐶𝐷𝐸</sub> = 𝑆<sub>𝐵𝐶𝐸𝐹</sub> + 𝑆<sub>𝐶𝐾𝐸</sub>+ 𝑆<sub>𝐶𝐷𝐸</sub> = 𝑆<sub>𝐵𝐶𝐸𝐹</sub>+ 𝑆<sub>𝐶𝐾𝐸𝐷</sub>
= 𝐵𝐶 ⋅ 𝐶𝐸 ⋅ sin 𝛼 +1


2⋅ 𝐶𝐸 ⋅ 𝐾𝐷 ⋅ sin 𝛼 = 𝐶𝐸 ⋅ sin 𝛼 +
1


2⋅ 2 ⋅ 𝐶𝐸 ⋅ sin 𝛼
= 2𝐶𝐷 ⋅ sin 𝛼


Mặt khác:


2√2 = 𝐷𝐶 + 𝐷𝐸 = 𝐸𝐶(sin ∠𝐷𝐶𝐸 + sin ∠𝐷𝐸𝐶) = 𝐸𝐶 (sin (𝛼 −𝜋


4) + sin (𝛼 +
𝜋
4))
= 2𝐸𝐶 ⋅ sin 𝛼 ⋅ cos𝜋


4


Suy ra 𝐸𝐶 ⋅ sin 𝛼 = 1, suy ra 𝑆𝐴𝐵𝐶𝐷𝐸𝐹 = 2.


C


E
K


A


B


F


</div>
<span class='text_page_counter'>(50)</span><div class='page_container' data-page=50>

<b>48. Về phía ngoài của tam giác 𝑨𝑩𝑪, ta lấy các điểm 𝑷, 𝑸, 𝑹 thỏa mãn các điều kiện: </b>
∠𝑷𝑨𝑩 = ∠𝑩𝑸𝑪 = 𝟒𝟓°, ∠𝑨𝑩𝑷 = ∠𝑸𝑩𝑪 = 𝟕𝟓°, ∠𝑪𝑨𝑹 = 𝟏𝟎𝟓°,𝑨𝑹


𝑨𝑪 = √
𝟑
𝟐
𝐓𝐢́𝐧𝐡 𝑹𝑸


𝑪𝑴


<i>Lời giải: </i>


Khơng mất tính tổng qt giả sử tam giác 𝐴𝐵𝐶 có hướng dương.
Gọi ℛ là phép quay vector góc +75°. Ta có:


ℛ(𝑄𝑅⃗⃗⃗⃗⃗ ) = ℛ(𝑄𝐵⃗⃗⃗⃗⃗ + 𝐵𝐴⃗⃗⃗⃗⃗ + 𝐴𝑅⃗⃗⃗⃗⃗ ) = ℛ(𝑄𝐵⃗⃗⃗⃗⃗ ) + ℛ(𝐵𝐴⃗⃗⃗⃗⃗ ) + ℛ(𝐴𝑅⃗⃗⃗⃗⃗ ) =𝑄𝐵



𝐵𝐶⋅ 𝐶𝐵⃗⃗⃗⃗⃗ +
𝐴𝐵


𝐵𝑃⋅ 𝐵𝑃⃗⃗⃗⃗⃗ +
𝐴𝑅
𝐴𝐶⋅ 𝐶𝐴⃗⃗⃗⃗⃗
Mặt khác:


𝑄𝐵
𝐵𝐶 =


𝐴𝐵
𝐵𝑃 =


sin 75°
sin 60° = √


3
2=


𝐴𝑅
𝐴𝐶
Suy ra:


ℛ(𝑄𝑅⃗⃗⃗⃗⃗ ) = √3


2(𝐶𝐵⃗⃗⃗⃗⃗ + 𝐵𝑃⃗⃗⃗⃗⃗ + 𝐶𝐴⃗⃗⃗⃗⃗ ) = √
3


2(𝐶𝑃⃗⃗⃗⃗⃗ + 𝐶𝐴⃗⃗⃗⃗⃗ ) = √6 ⋅ 𝐶𝑀⃗⃗⃗⃗⃗⃗


Vậy 𝑅𝑄


𝐶𝑀= √6.


<b>49. Cho tam giác 𝑨𝑩𝑪 cân tại 𝑨. Các điểm </b>𝑴, 𝑵 thuộc cạnh 𝑨𝑩. Các điểm 𝑷, 𝑸 thuộc cạnh
𝑨𝑪. 𝑺 ≡ 𝑴𝑷 ∩ 𝑵𝑸. Các đường tròn (𝑴, 𝑴𝑩) và (𝑷, 𝑷𝑪) cắt nhau tại 𝑿, 𝒀. Các đường tròn
(𝑵, 𝑵𝑩) và (𝑸, 𝑸𝑪) cắt nhau tại 𝒁, 𝑻. Chứng minh rằng, 𝑺𝑿 = 𝑺𝒀 = 𝑺𝒁 = 𝑺𝑻.


I
M


N


R


Q


P


A
B


</div>
<span class='text_page_counter'>(51)</span><div class='page_container' data-page=51>

<i>Lời giải:</i>


Gọi 𝐴𝐷 là đường kính của đường trịn (𝐴𝐵𝐶). Ta có 𝐷𝐵 ⊥ 𝐴𝐵, 𝐷𝐶 ⊥ 𝐴𝐶 nên 𝐷𝐵 tiếp xúc với
(𝑀, 𝑀𝐵) và (𝑁, 𝑁𝐵) còn 𝐷𝐶 tiếp xúc với (𝑃, 𝑃𝐶) và (𝑄, 𝑄𝐶). Từ đó ta có:


𝒫<sub>𝐷,(𝑀,𝑀𝐵)</sub> = 𝒫<sub>𝐷,(𝑁,𝑁𝐵)</sub>= 𝐷𝐵2 = 𝐷𝐶2 = 𝒫<sub>𝐷,(𝑃,𝑃𝐶)</sub> = 𝒫<sub>𝐷,(𝑄,𝑄𝐶)</sub>


Suy ra 𝐷 là tâm đẳng phương của bốn đường trịn nói trên. Vì thế, 𝐷 ≡ 𝑋𝑌 ∩ 𝑍𝑇. Suy ra:


𝐷𝑋


̅̅̅̅ ⋅ 𝐷𝑌̅̅̅̅ = 𝐷𝑍̅̅̅̅ ⋅ 𝐷𝑇̅̅̅̅


Vậy 𝑋, 𝑌, 𝑍, 𝑇 đồng viên. Mặt khác 𝑀𝑃, 𝑁𝑄 là trung trực của 𝑋𝑌, 𝑍𝑇 nên 𝑆 là tâm của đường
tròn (𝑋𝑌𝑇𝑍), tức là 𝑆𝑋 = 𝑆𝑌 = 𝑆𝑍 = 𝑆𝑇 (đpcm).


<b>50. Cho tứ giác 𝑨𝑩𝑪𝑫 có các cạnh đối khơng song song và 𝑨𝑪 = 𝑩𝑫. Đường trung trực của </b>
<b>các cạnh </b>𝑨𝑩, 𝑪𝑫 cắt nhau tại điểm 𝑬 nằm trong tứ giác. Đường trung trực của các cạnh
𝑨𝑫, 𝑪𝑩 cắt nhau tại điểm 𝑭 nằm trong tứ giác.


X


Y


T
Z
S


D
A


B C


N
M


</div>
<span class='text_page_counter'>(52)</span><div class='page_container' data-page=52>

<b>a. </b> <b>Chứng minh rằng ∠𝑨𝑬𝑩 + ∠𝑨𝑭𝑫 = 𝟏𝟖𝟎°. </b>


<b>b. </b> <b>Chứng minh rằng 𝑬𝑭 đi qua trọng tâm của tứ giác. </b>



<i>Lời giải:</i>


Gọi 𝐼, 𝐽 là trung điểm của 𝐴𝐶, 𝐵𝐷. Theo giả thiết ta có 𝐸𝐴 = 𝐸𝐵, 𝐸𝐶 = 𝐸𝐷, 𝐹𝐵 = 𝐹𝐶, 𝐹𝐴 = 𝐹𝐷
và 𝐴𝐶 = 𝐵𝐷 nên các cặp tam giác 𝐸𝐴𝐶, 𝐸𝐵𝐷 và 𝐹𝐴𝐶, 𝐹𝐵𝐷 bằng nhau thuận. Từ đó:


(𝐸𝐴, 𝐸𝐵) ≡ (𝐴𝐶, 𝐵𝐷) ≡ (𝐹𝐷, 𝐹𝐴) (mod 𝜋)
Suy ra ∠𝐴𝐸𝐵 + ∠𝐴𝐹𝐵 = 180° (đpcm).


Mặt khác cũng từ các cặp tam giác bằng nhau này ta suy ra 𝐸𝐼 = 𝐸𝐽 và 𝐹𝐼 = 𝐹𝐽. Vậy 𝐸𝐹 đi qua
trung điểm 𝐺 của 𝐼𝐽, chính là trọng tâm của tứ giác 𝐴𝐵𝐶𝐷 (đpcm).


<b>51. Cho tứ giác 𝑨𝑩𝑪𝑫 nội tiếp có 𝑨𝑩 ⋅ 𝑩𝑪 = 𝟐𝑨𝑫 ⋅ 𝑫𝑪. Chứng minh rằng: 𝟖𝑩𝑫</b>𝟐<sub>≤ 𝟗𝑨𝑪</sub>𝟐<b><sub>. </sub></b>


<i>Lời giải:</i>


G


J
I


F
B


D
C


A


</div>
<span class='text_page_counter'>(53)</span><div class='page_container' data-page=53>

Ta có:



𝐵𝐴 ⋅ 𝐵𝐶 = 2𝐷𝐴 ⋅ 𝐷𝐶 ⇒ 𝐵𝐴 ⋅ 𝐵𝐶 ⋅ sin ∠𝐴𝐵𝐶 = 2𝐷𝐴 ⋅ 𝐷𝐶 ⋅ sin ∠𝐴𝐷𝐶 ⇒ 𝑆<sub>𝐵𝐴𝐶</sub> = 2𝑆<sub>𝐷𝐴𝐶</sub>
⇒𝐼𝐵


𝐼𝐷=
𝑆<sub>𝐴𝐼𝐵</sub>
𝑆<sub>𝐴𝐼𝐷</sub>=


𝑆<sub>𝐶𝐼𝐵</sub>
𝑆<sub>𝐶𝐼𝐷</sub> =


𝑆<sub>𝐴𝐼𝐵</sub>+ 𝑆<sub>𝐶𝐼𝐵</sub>
𝑆<sub>𝐴𝐼𝐷</sub>+ 𝑆<sub>𝐶𝐼𝐷</sub> =


𝑆<sub>𝐵𝐴𝐶</sub>


𝑆<sub>𝐷𝐴𝐶</sub> = 2 ⇒ 𝐼𝐵 =
2𝐵𝐷


3 ; 𝐼𝐷 =
𝐵𝐷


3
Theo bất đẳng thức Cauchy ta có:


2𝐵𝐷2


9 = 𝐼𝐵 ⋅ 𝐼𝐷 = 𝐼𝐴 ⋅ 𝐼𝐶 ≤


(𝐼𝐴 + 𝐼𝐶)2



4 =


𝐴𝐶2


4 ⇒ 8𝐵𝐷


2 <sub>≤ 9𝐴𝐶</sub>2


Vậy ta có đpcm.


Đẳng thức xảy ra khi và chỉ khi 𝐼𝐴 = 𝐼𝐶, ví dụ như khi 𝐵𝐷 là đường kính của (𝑂) và 𝐴𝐶 đi qua
𝐼 ∈ 𝐵𝐷 sao cho 𝐼𝐵 = 2𝐼𝐷.


<b>52. Về phía ngoài tam giác </b>𝑨𝑩𝑪 lấy các điểm 𝑿, 𝒀 sao cho các tam giác 𝑨𝑩𝑿, 𝑨𝑪𝒀 đồng
<b>dạng ngược hướng. Lấy các điểm </b>𝒁, 𝑻 sao cho các tam giác 𝑩𝒁𝑪, 𝑩𝑿𝑨, 𝑻𝑿𝒀 đồng dạng
<b>cùng hướng. Chứng minh rằng, các tam giác 𝑩𝒁𝑪, 𝑻𝑿𝒀 có cùng tâm đường tròn ngoại tiếp. </b>


<i>Lời giải:</i>


I
O


B


A


D


</div>
<span class='text_page_counter'>(54)</span><div class='page_container' data-page=54>

Từ giả thiết ta thấy rằng:



Δ𝐵𝑍𝑋 ∼ Δ𝐵𝐴𝐶 ⇒𝑋𝑍
𝐴𝐶 =


𝐵𝑋
𝐵𝐴⇒


𝑋𝑍
𝐵𝑋 =


𝐴𝐶
𝐴𝐵
Δ𝑋𝐵𝑇 ∼ Δ𝑋𝐴𝑌 ⇒𝐵𝑇


𝐴𝑌 =
𝐵𝑋
𝐴𝑋⇒


𝐵𝑇
𝐵𝑋 =


𝐴𝑌
𝐴𝑋
Δ𝐴𝐵𝑋 ∼ Δ𝐴𝐶𝑌 ⇒ 𝐶𝑌


𝐵𝑋 =
𝐴𝐶
𝐴𝐵 =


𝐴𝑌


𝐴𝑋
Từ các điều trên suy ra:


𝑋𝑍
𝐵𝑋 =


𝐴𝐶
𝐴𝐵 =


𝐴𝑌
𝐴𝑋 =


𝐵𝑇
𝐵𝑋 =


𝐶𝑌


𝐵𝑋 ⇒ 𝑋𝑍 = 𝐵𝑇 = 𝐶𝑌


Mặt khác, gọi 𝑂 là tâm của phép đồng dạng thuận ℋ: 𝑋 ↦ 𝑍, 𝑇 ↦ 𝐵, 𝑌 ↦ 𝐶 thì ta có:
{Δ𝑂𝑍𝑋 ∼ Δ𝑂𝐵𝑇 ∼ Δ𝑂𝐶𝑌


𝑋𝑍 = 𝐵𝑇 = 𝐶𝑌 ⇒ Δ𝑂𝑍𝑋 = Δ𝑂𝐵𝑇 = Δ𝑂𝐶
Suy ra 𝑂𝑍 = 𝑂𝐵 = 𝑂𝐶 và 𝑂𝑋 = 𝑂𝑌 = 𝑂𝑇.


Vậy (𝐵𝑍𝐶) và (𝑇𝑋𝑌) có cùng tâm đường trịn ngoại tiếp là 𝑂 (đpcm).


T


O


Z


Y
A


B <sub>C</sub>


</div>

<!--links-->

×